You are on page 1of 84

File Downloaded from www.carocks.wordpress.

com

TAXATION
Amendments made by Finance Act, 2012 (For A.Y. 2013-14) Relevant for Nov. 2013 Exams.

CA. Krishna Shankar Prasad


ICAI Faculty Member

Basic Concepts
Page 7

F Rates of Income Tax for Assessment Year 2013-14 [In the Case of Individual/HUF]
1. For an individual (man or woman), resident in India who is of the age of 60 years or more but less then 80 years at any time during the previous year [Senior Citizen] Upto ` 2,50,000 Next ` 2,50,000 Next ` 5,00,000 Balance Income 2. Nil 10% 20% 30%

For an individual (man or woman), resident in India who is of the age of 80 years or more at any time during the previous year [Very Senior Citizen] Upto ` 5,00,000 Next ` 5,00,000 Balance Income Nil 20% 30% Nil 10% 20% 30%

3.

Other Individual / HUF / AOP / Body of Individuals :Upto ` 2,00,000 Next ` 3,00,000 Next ` 5,00,000 Balance Income Education cess : Education cess of 2% shall be levied on the Income tax. Secondary and Higher Education cess :- 1% shall be levied on the Income tax.

Surcharge : No surcharge is payable as it has been abolished for above persons.

KSP Classes; PH : 9811880881, 9868308387

[ Amendment Page - 1]

CA. Krishna Shankar Prasad

Rates of Income Tax for A.Y. 2013-14


Amount of Tax
@ 30% Firms/LLP Local Authority @ 30% Upto ` 10,000 @ 10% Co-operative Next ` 10,000 @ 20% Societies Balance Income @ 30% Domestic Company @ 30% Companies Foreign Company @ 40%. Nil Nil Nil

Surcharge

@ 5% on tax payable shall be levied where total income of the co. exceeds one crore rupees. @ 2% on tax payable shall be levied where total income of the foreign company exceeds one crore rupees.

For all assessee - Education Cess @ 2%, and Secondary and Higher Education Cess @ 1% on income tax (inclusive of surcharge if applicable) shall be chargeable.

F
(1)

Deemed Incomes :Unexplained Cash Credits (Section 68) The amount credited in the books of assessee. He offers no explanation about its nature and source or the explanation offered is not satisfactory. The amount credited is treated as the income of the previous year in which books of accounts found credited. Unexplained cash credit shall be chargeable to tax under the head Income from Other Sources. Where the assessee is a closely held company and the sum so credited consists of share application money, share capital, share premium or any such amount then the person being a resident in whose name such credit is recorded in the books of such company should offer an explanation about the nature and source of such sum credited. If the nature and source of such amount is not found satisfactory by the Assessing Officer then such amount credited shall be treated as income of the company in whose books of account such sum is found credited. Unexplained Investments (Section 69) The assessee made investments but has not recorded in his books. He offers no explanation about its nature and source or the explanation offered is not satisfactory. The value of the investment made shall be treated as the income of that financial year in which investment is made. Unexplained Investments shall be chargeable to tax under the head Income from Other Sources. Unexplained Money, bullion or jewel or Valuable article (Section 69A) In search, the assessee was found to be owner of any money, bullion or jewel or other valuable article etc. Such money, bullion etc. are not recorded in the books of accounts of the assessee. He offers no explanation about its nature and source of acquisition or the explanation offered is not satisfactory.

(2)

(3)

KSP Classes; PH : 9811880881, 9868308387

[ Amendment Page - 2]

CA. Krishna Shankar Prasad


The value of such items shall be treated as the income of that financial year it is found. Unexplained Money, Bullion, Jewelry etc. shall be chargeable to tax under the head Income from Other Sources.

(4)

Investment not fully disclosed (Section 69B) The assessee made investments or found to be owner of bullion, jewellery or other valuable article, but has not fully recorded in his books of accounts. He offers no explanation about such excess amount or the explanation offered is not satisfactory. The value of the investment made shall be treated as the income of that financial year in which investment is made. The excess value of unexplained investments shall be chargeable to tax under the head Income from other sources. Unexplained Expenditure (Section 69C) The assessee has incurred expenditure during the financial year. He offers no explanation about such expenditure or the explanation offered is not satisfactory. The amount of such expenditure shall be treated as income of the previous year in which it was incurred. Unexplained expenditure shall be chargeable to tax under the head Income from Other Sources. It shall not be allowed as a deduction under any head of income. Amount borrowed or repaid on hundi, other than by way of account payee cheque (Section 69D) The amount is borrowed on a Hundi or the same was repaid in excess of ` 20,000. The borrowal is made otherwise than through an Account Payee Cheque. The amount is borrowed or repaid shall be treated as income of the person borrowing or repaying the amount for the previous year in which the amount was borrowed or repaid. Such amount borrowed or repaid shall be chargeable to tax under the head Income from Other Sources. Any amount borrowed on Hundi and treated as income u/s 69D shall not be taxed once again at the time of repayment. Undisclosed income referred to in section 68, 69, 69A, 69B, 69C or 69D to be taxable at special rate [w.e.f. A.Y. 2013-14] Rate of Tax @ 30%. (without aallowing any deduction or expenditure).

(5)

(6)

KSP Classes; PH : 9811880881, 9868308387

[ Amendment Page - 3]

CA. Krishna Shankar Prasad

Profits and Gains of Business of Profession


F
(4)

Additional Depreciation Section 32(1)(iia) :Power generation equipments qualify for claiming additional depreciation in respect of new plant and machinery from A.Y. 2013-14

Investment-linked tax incentive for specified business Section 35AD :(i) 100%, deduction would be allowed in respect of the whole of any expenditure of capital nature incurred, wholly and exclusively, for the purposes of the specified business during the previous year in which such expenditure is incurred. However, the expenditure incurred, wholly and exclusively, for the purpose of any specified business shall be allowed as deduction during the previous year in which he commences operations of his specified business, if (a) the expenditure is incurred prior to the commencement of its operations; and (b) the amount is capitalised in the books of account of the assessee on the date of commencement of its operations. (ii) The expenditure of capital nature shall not include any expenditure incurred on acquisition of any land or goodwill or financial instrument. (iii) The benefit will be available Specified business Date of Commencement of business On or after 1-4-2009. On or after 1-4-2007, in the case of laying and operating & a crosscountry natural gas pipeline network for distribution or storage. In other cases, on or after 1-4-09. On or after 1-4-2010.

Setting up and operating a cold chain facility, warehousing facility for storage of agricultural produce. Laying and operating a cross-country natural gas or crude or petroleum oil pipeline network for distribution, including storage facilities being an integral part of such network

Business of building and operating anywhere in India a new hotel of 2 star or above category, a new hospital with at least 100 beds for patients, developing and building a housing project under a scheme for slum redevelopment or rehabilitation. Developing and Building a housing project & production of fertilizer in India Setting up ICD or CFS, Bee-keeping, warehousing facility for storage of sugar

On or after 1-4-2011. On or after 1-4-2012.

(iv) This section applies to the specified business which fulfils all the following conditions, namely :(a) It is not set up by splitting up, or the reconstruction, of a business already in existence; (b) It is not set up by the transfer to the specified business of machinery or plant previously used for any purpose. However, 20% value of the total plant and machinery can be second hand.

KSP Classes; PH : 9811880881, 9868308387

[ Amendment Page - 4]

CA. Krishna Shankar Prasad


(c) Where the assessee is carrying on business relates to laying and operating a cross country natural gas or petroleum oil pipeline network for distribution then such business : (i) is owned by a company formed and registered in India under the Companies Act, 1956 or by a consortium of such companies or by an authority or a board or a corporation established or constituted under any Central or State Act; (ii) has been approved by the Petroleum and Natural Gas Regulatory Board; (iii) has made not less than one-third (for a natural gas pipeline network) or one-fourth (for petroleum product pipeline network) of its total pipeline capacity available for use on common carrier basis by any person other than the assessee or an associated person. (iv) fulfils any other condition as may be prescribed.

Key Note :(a) an associated person, in relation to the assessee, means a person, (i) (ii) who participates in the management of the assessee; who holds at least 26% voting power in the assessee;

(iii) who appoints more than half of the Board of directors; or (iv) who guarantees not less than 10% of the total borrowings of the assessee; (b) cold chain facility means a chain of facilities for storage or transportation of agricultural and forest produce, meat and meat products, poultry, marine and dairy products, products of horticulture, floriculture and apiculture and processed food items under scientifically controlled condition including refrigeration and other facilities necessary for the preservation of such produce; specified business means the any one or more of the following business, namely (i) setting up and operating a cold chain facility; (ii) setting up and operating a warehousing facility for storage of agricultural produce; (iii) laying and operating a cross-country natural gas or crude or petroleum oil pipeline network for distribution, incluing storage facilities being an integral part of such network. (iv) Business of building and operating anywhere in India a new hotel of 2 star or above category (v) A new hospital with at least 100 beds for patients, (vi) Developing and building a housing project under a scheme for slum redevelopment or rehabilitation. (vii) Developing and building a housing project under a scheme for affordable housing framed by the Central Government or a State Government, as the case may be, and notified by the Board in this behalf in accordance with the guidelines as may be prescribed (viii) production of fertiliser in India. (ix) setting up and operating an Inland Container Depot or Container Freight Station notified and approved under the Customs Act, 1962; (x) bee-keeping and production of honey and beeswax; and (xi) setting up and operating a warehousing facility for storage of sugar.

(c)

KSP Classes; PH : 9811880881, 9868308387

[ Amendment Page - 5]

CA. Krishna Shankar Prasad Nature and amount of deduction [Section 35AD] :
100% deduction shall be allowed an account of any expenditure of capital nature incurred wholly and exclusively for the purpose of the above specified business carried on by such assessee during the previous year in which such expenditure in incurred by him. However, in case any of the following business has commenced its operation on or after 1-4-2012, the deduction allowed shall be 150% of such capital expenditure incurred instead of 100%. (i) Setting up and operating a cold chain facility. (ii) Setting up and operating a warehouse for storage of agricultural produce. (iii) Building and operating, anywhere in India, a hospital with atleast one hundred beds for patients. (iv) Developing and building a housing project under a scheme for affordeable housing framed by the Central Government or State Government and notified by the Board in accordance with guidelines as prescribed. (v) Production of fertilizer in India.

Expenditure incurred on agricultural extension project [Section 35CCC] [W.e.f. A.Y. 2013-14]
Nature of Expenditure :- Expenditure incurred on agricultural extension project.. Deduction :- Weighted deduction @ 150% Conditions: - Where a deduction claimed and allowed for any assessment year in respect of any expenditure referred to in section 35CCC(1), deduction shall not be allowed in respect of such expenditure under any other provisions of the Income-tax Act for the same or any other assessment year.

Expenditure incurred by a company on skill development project [Section 35CCD] [W.e.f. A.Y. 2013-14]
Nature of Expenditure :- Expenditure incurred by a company on skill development project..(not being expenditure in the nature of cost of any land or building). Deduction :- Weighted deduction @ 150% Conditions: - Where a deduction claimed and allowed for any assessment year in respect of any expenditure referred to in section 35CCD(1), deduction shall not be allowed in respect of such expenditure under any other provisions of the Income-tax Act for the same or any other assessment year.

KSP Classes; PH : 9811880881, 9868308387

[ Amendment Page - 6]

CA. Krishna Shankar Prasad F

Expenses not Deductible Section 40(a) :Nature of expenditure (1) Salary, interest, royalty, fees for technical services payable outside India or in india to nonresident on which tax has not been paid or deducted at sources. u Will be allowed as a deduction in computing the income of the previous year in which such tax has been paid. (2) Interest, Commission or brokerage, rent, fees for technical services, Fees for professioanl services, Royalty, Payment to contractors / sub-contractors payable to resident on which tax has not been paid or deducted at source. Expenditure - Is Is such expenditure deductible in it deductible in any subsequent previous year the current previous year Case 1 - Tax is No deduction in the If tax is deducted in any subsequent year, deductible but not current previous year the expenditure will be deducted in the year in which TDS will be deposited by the deducted assessee with the Government. Case 2 - Tax is No deduction in the If tax is deposited with the Government deductible (and is so current previous year after the due date of submission of return of income the expenditure will be deducted) during the deductible in that year in which tax will be current Financial Year deposited. but it is not deposited on or before the due date of submission of return of income under section 139(1). Cases (3) Rates of taxes levied on profits or gains of any business or profession u Not deductible (4) Wealth Tax u Not deductible

Circumstances under which tax shall be deemed to have been deducted and paid Where an assessee makes the payment of an expense to a resident payee without deduction of tax and is not deemed to be an assessee in default under section 201(1) because the payee and the payer have satisfied conditions mentioned therein then it shall be deemed that the assessee has deducted & paid the tax on such sum on the date of furnishing of return of income by the resident payee. Conditions to be satisfied by the resident payee (i) he has furnished his return of income under section 139; (ii) he has taken into account such sum for computing income in such return of income; (iii) has has paid the tax due on the income declared by him in such return in income. Conditions to be satisfied by the payer (i.e. assessee) The payer furnishes a certificate to this effect from a Chartered Accountant in such form as may be prescribed.
KSP Classes; PH : 9811880881, 9868308387 [ Amendment Page - 7]

CA. Krishna Shankar Prasad F


(1)

Compulsory Audit of Accounts Section 44AB :Applicability in the case of :(a) Assessee carrying on any business where total sales or gross receipts exceeds ` 1,00,00,000; or (b) Assessee carrying on profession where gross receipts exceeds ` 25,00,000; or (c) Assessee carrying on business referred to u/s 44AD/AE/BB/BBB and declaring lower income than prescribed. Filing of report :(i) Where the assessee has under taken any international transaction as per section 92B or specified domestic transaction as per section 92BA (ii) In any other case 30th November of the relevant assessment year

(2)

30th September of the relevant assessment year

(3)

Other Point :If accounts are required to be audited under any other law, then it will be sufficient compliance if the assessee get the accounts audited under such law and furnish an audit report by the due date under such other law and a further report under this section. Consequence of non-compliance :If the audit report is not submitted alongwith return then AO may treat the return as defective return.

(4)

Presumptive income in case of Specific Business or Profession Section 44AD/AE :Particulars Section 44AD Eligible business Any business except the business of section 44AE (Gross receipt does not exceeds ` 1 crore) Eligible assessee Individual, HUF, Firm (not being a LLP) M i n i m u m 8% or more of amount of gross turnover or p r e s u m p t i v e gross receipts income Section 44AE Persons carrying on business of plying, hiring and leasing goods carriages and not owning more than 10 goods carriages at any time during the P/Y. Any Assessee

In case of Heavy goods vehicle - ` 5,000 per month or part of a month. Others - ` 4,500 per month or part of a month Income shall be considered from the date of ownership.
[ Amendment Page - 8]

KSP Classes; PH : 9811880881, 9868308387

CA. Krishna Shankar Prasad

Common Provisions in case of all the above two Sections :(1) Allowability of deduction u/s 30 to 38 --- Deemed to be allowed (2) Allowability of depreciation in case of subsequent previous year--- WDV of the assets shall be computed as if depreciation had been allowed in earlier years. (3) Consideration of turnover for Compulsory audit u/s 44AB shall not be considered if the assessee opts for section 44AD/AE. (4) Option for lesser amount of Income --- Books of accounts to be maintained u/s 44AA and audit u/s 44AB has to be fulfilled. (5) Computation of presumptive income in case the assessee is a firm --- Compute presumptive income as above Less interest, salary and remuneration to partners. The balance is chargeable to tax. (6) Deduction u/s 80C to 80U shall be available to the assessee. (7) The provision of section 44AD shall not apply to (a) a person carrying on specified profession as referred to in section 44AA(1), (b) a person earning income in the nature of commission or brokerage, or (c) a person carrying on any agency business.

Capital Gains
F Cost Inflation Index (CII) as notified by Central Government is as under :Financial Year 2012-13 852

Fair Market Value to be full value of consideration in certain cases Section 50D [W.e.f. A.Y. 2013-14]:
Where the consideration received of accruing as a result of the transfer of a capital asset by an assessee :(a) is not ascertainable, or (b) cannot be determined. then, for the purpose of computing income chargeable to tax as capital gains, the fair market value of the said asset on the date of transfer shall be deemed to be the full value of the consideration received or accruing as a result of such transfer.

KSP Classes; PH : 9811880881, 9868308387

[ Amendment Page - 9]

CA. Krishna Shankar Prasad F

Exemption of long-term capital gains tax on transfer of residential property if invested in a new manufacturing SME company Section 54GB [W.e.f. A.Y. 2013-14]
(a) Asset Transferred : Residential Property (a house or plot of land) (b) Who is entitled : Individual or HUF (c) Use or Holding period : Exceeding 36 months (d) Amount to be invested : Net consideration (e) New Asset : The amount of net consideration should be utilized by the individual or HUF before the due date of furnishing of return of income under section 139(1), for subscription in equity shares of a eligible company The amount of subscription as share capital is to be utilized by the company for the purchase of new asset (eligible plant and machinery) within a period of one year from the date of subscription in the equity shares. (f) Treatment of unutilised amount by Company : If the amount of net consideration which has been received by the company for the issue of equity shares by the individual of HUF is not utilized by the company for the purchase of a new asset (eligible plant and machinery) before the due date of furnishing the return of income under section 139, the unutilized amount should be deposited before the said due date under a deposit scheme, notified by the Central Government in this behalf and the return furnished by the assessee shall be accompanied by proof of such deposit having been made. (g) Consequences if the amount deposited in deposit scheme is not utilised, wholly or party for the purchase of new asset within a period of one year from the date of subscription in equity shares by the Individual or HUF :- Taxable as a long-term capital gain in the hands of individual or HUF. Taxable Amount = The exemption allowed under section 54GB less the exemption that should have been allowed based on the amount actually utilized, in the purchase of new asset (eligible plant and machinery). (h) Consequence if equity shares or new asset is transferred within a period of 5 years from the date of its acquisition : (i) If the equity shares acquired by the individual or HUF are sold or otherwise transferred within a period of 5 years from the date of its acquisition, the capital gain shall arise as under: (a) the capital gain arising from the transfer of equity shares shall be taxable in the previous year in which such shares are transferred, which can be shortterm or long-term depending upon the period of holding. (b) the amount of capital gain which was exempt under section 45(1) earlier shall be taxable as long-term capital gain of the previous year in which such shares are sold or otherwise transferred and hence taxable.

KSP Classes; PH : 9811880881, 9868308387

[ Amendment Page - 10]

CA. Krishna Shankar Prasad


(ii) Similarly, if the new asset (eligible plant and machinery) is sold or otherwise transferred by the company within a period of 5 years from the date of its acquisition, the capital gain shall arise as under: (a) the capital gain, if any arising from the transfer of such asset will be taxable in the hands of company, which will be sort-term as asset is a depreciable asset forming part of block of asset. (b) the amount of capital gain which was exempt under section 45(1) earlier shall be taxable as long-term capital gain in the hands of such individual or HUF in the previous year in which such asset (eligible plant and machinery) is sold or otherwise transferred.

Other Sources
Share premium in excess of the Fair Market value to be treated as Income[Section 56(2)(viib)] [W.e.f. A.Y. 2013-14] :Where a company, not being a company in which the public are substantially interested, receives, in any previous year, from any person being a resident, any consideration for issue of shares and if the consideration received for issue of shares exceeds the face value of such shares, the aggregate consideration received for such shares as exceeds the fair market value of the shares shall be chargeable to income-tax under the head Income from other sources. However, the above provision shall not apply where the consideration for issue of shares is received (i) by a venture captial undertaking from a venture capital company or a venture capital fund; or (ii) by a company from a class or classes of persons as may be notified by the Central Government in this behalf.

Deductions
F

Deduction in respect of Life Insurance Premium contribution to Provident Fund, etc. Section 80C :Applicability :- Only Individual and HUF Quantum of Deduction :- Maximum ` 1,00,000 Nature of Payment :- On the following amounts deduction is allowed (i) Life Insurance Premium on life of himself, Spouse and Children, in HUF any member of family. (Maximum 10% of the Capital Sum assured).
[ Amendment Page - 11]

KSP Classes; PH : 9811880881, 9868308387

CA. Krishna Shankar Prasad

F Deduction in respect of investment made under an equity savings scheme Section 80CCG [W.e.f. A.Y. 2013-14]
Applicability: Individual who is resident in India. Nature of Payment : Deduction is allowed to such assessee who has (i) in a previous year, acquired listed equity shares in accordance with a scheme, as may be notified by the Central Government in this behalf, and (ii) satisfied the prescribed conditions. Quantum of Deduction : (i) 50% of the amount invested in such equity shares (ii) ` 25,000 whichever is less. Condition to the satisfied : (i) The gross total income of the assessee for the relevant assessment year should not exceed ` 10,00,000; (ii) The assessee is a new retail investor as may be specified under the scheme notified in this behalf; (iii) The investment is made in such listed equity shares as may be specified under the notifiedd scheme; (iv) The investment is locked-in for a period of three years from the date of acquisition in accordance with the notified scheme; and (v) Such other condition as may be prescribed. Consequences if the above condition are not satisfied : If the assessee, in any previous year, fails to comply with any condition specified above, the deduction originally allowed shall be deemed to be the income of the assessee of such previous year and shall be liable to tax for the assessment year relevant to such previous year.

KSP Classes; PH : 9811880881, 9868308387

[ Amendment Page - 12]

CA. Krishna Shankar Prasad


F

Deduction of Medical Insurance Premium Section 80D :Applicability :- Individual or HUF. Nature of payment :- Mediclaim insurance policy, any contribution made to the Central Govt. Health Scheme (CGHS) and payment made on account of preventive Health Check-up (Maximum ` 5,000) :(1) In case of individual, in the name of individual, his spouse, parents (whether dependent or not) and dependent children. (2) In case of HUF, in the name of any member. Quantum of deduction :Individual Taxpayer, spouse and dependent children ` 15,000 5,000 Parents of the taxpayer whether dependent or not ` 15,000 5,000 HUF Any member of family ` 15,000 5,000

Maximum amount deductible (including preventive Health Check-up) Additional amount which is deductible when policy is taken on the health of a senior citizen (i.e., resident in India and 60 years or more)

Conditions :(i) Premium can be paid by any mode other than cash. However for preventive Health Check-up it can be made in cash also. (ii) Payment shall be made out of income chargeable to tax. (iii) Payment as per GIC scheme or any other insurance as approved by Central Govt.

D e d u c t i o n in respect of donation etc. Section 80G :Key Notes :(1) (i) Qualifying limit :- 10% of Adjusted Gross Total Income (ii) Adjusted Gross Total Income means :Gross Total Income Less : (i) Deduction u/s 80C to 80U except u/s 80G (ii) LTCG ( if already included in GTI) (iii) STCG u/s 111A (2) Donation in kind are not eligible. (3) No deduction shall be allowed under this section in respect of donation of any sum exceeding ` 10,000 unless such sum is paid by any mode other than cash.

KSP Classes; PH : 9811880881, 9868308387

[ Amendment Page - 13]

CA. Krishna Shankar Prasad


F

D e d u c t i o n in respect of donation for scientific research or rural development Section 80GGA :(vi) No deduction shall be allowed under section 80GGA in respect of any sum exceeding ` 10,000 unless such sum is paid by any mode other than cash.

Deduction in respect of interest on deposits in savings accounts to the maximum extent of ` 10,000 Section 80TTA [W.e.f. A.Y. 2013-14]
Where the gross total income of an assessee, being an individual or a Hindu undivided family, includes any income by way of interest on deposits (not being time deposits) in a savings account with (a) a banking company to which the Banking Regulation Act, 1949, applies (including any bank or banking institution referred to in section 51 of that Act); (b) a co-operative society engaged in carrying on the business of banking (including a cooperative land mortgage bank or a co-operative land development bank); or (c) a Post Office Quantum of Deduction :- a deduction of such interest shall be allowed to the maximum extent of ` 10,000. Key Note: Where the income referred to in this section is derived from any deposit in a savings account, held by, or on behalf of, a firm, an association of persons or a body of individuals, no deduction shall be allowed under this section in respect of such income in computing the total income of any partner of the firm or any member of the association or any individual of the body.

KSP Classes; PH : 9811880881, 9868308387

[ Amendment Page - 14]

CA. Krishna Shankar Prasad

Return of Income
Compulsory filing of income tax return in relation to assets located outside India [Fourth proviso to section 139(1)] [W.r.e.f. A.Y. 2012-13] Fourth proviso has been inserted to section 139(1) to provide that a person, being a resident, who is not required to furnish a return under section 139(1) and who during the previous year has : (a) any asset (including any financial interest in any entity) located outside India or (b) signing authority in any account located outside India, shall furnish, on or before the due date, a return in respect of his income or loss for the previous year in such form and verified in such manner and setting forth such other particulars as may be prescribed.

Tax Deducted at Source


F

TDS from Salary Section 192 :(1) (2) (3) (4) (5) Who is liable to deduct tax Type of Recipient Nature of Payment Rate of TDS Exemption Employers Employee Salary Rates of tax as applicable to the individual No TDS If income under head Salaries does not exceed u ` 5,00,000 (Resident very senior citizen), u ` 2,50,000 (Resident Senior Citizen), u ` 2,00,000 (Other individual), At the time of Payment In case of Government :- Same day In case of others :- One week from the last day of the month Form No. 16 along with Form No. 12BA Employee shall make an application in Form No. 13 to A.O. A.O. shall issue certificate in Plain Paper. The same is to be furnished to the employer.

(6) (7) (8) (9)

When TDS is to be deducted Time for deposit of TDS Certificate of TDS D eduction of Tax at lower rate or non-deduction of Tax

KSP Classes; PH : 9811880881, 9868308387

[ Amendment Page - 15]

CA. Krishna Shankar Prasad F

TDS from Payment to non-resident sportsmen or sports associations Section 194E :(1) (2) (3) (4) (5) (6) (7) Who is liable to deduct tax Type of Recipient Nature of Payment Rate of TDS Exemption When TDS is to be deducted Time for deposit of TDS Any Person Non-resident sportsmen / Sports association or non-resident & non-citizen entertainer Payment to non-resident Sportsman / Sports association 10% [W.e.f. 1.7.2012 @ 20%] plus EC @ 2% & SHEC @ 1% Nil At the time of payment or credit, whichever is earlier. In case of Government :- Same day. In all other cases : u Where income or amount is paid or credited in the month of March : 30th April. u Other cases - within 7 days from the last day of the month. Form 16A No Provision

(8) (9)

Certificate of TDS Deduction of Tax at lower rate or non-deduction of Tax

TDS on fees for professional or technical services Section 194J :(1) Who is liable to deduct tax All assessees except individual and HUF, who are not subject to tax audit u/s 44AB of the Act, during the preceding financial year. A resident person Fees for Professional Services, Fees for Technical Services, An remuneration or fee or commission by whatever name called, other than those or which tax is deductible under section 192, to a director for a company (W.e.f. 1.7.2012) Royalty, Sum received for not carrying out any activity or not sharing any know-how, patient etc. 10% Aggregate of payments does not exceed ` 30,000 in a financial year, for each of the nature of payment referred above. However there is no threshold limit of ` 30,000 in case a payment to a Director remuneration

(2) (3)

Type of Recipient Nature of Payment

(4) (5)

Rate of TDS Exemption

KSP Classes; PH : 9811880881, 9868308387

[ Amendment Page - 16]

CA. Krishna Shankar Prasad F

TDS on Compensation on acquisition of certain immovable properties Section 194LA :(1) (2) (3) Who is liable to deduct tax Type of Recipient Nature of Payment Any Person A resident person Paying compensation or enhanced compensation on account of compulsory acquisition under any law for any immovable property other than agricultural land. 10% If the amount of payment does not exceed ` 1,00,000 during a financial year. (W.e.f. 1.7.2012 ` 2,00,000) At the time of credit or payment, whichever is earlier. In case of Government :- Same day In case of others :- One week from the last day of the month Form 16A Application in Form No. 13 shall be made to the A.O. A.O. shall issue certificate in plain paper.

(4) (5) (6) (7) (8) (9)

Rate of TDS Exemption When TDS is to be deducted Time for deposit of TDS Certificate of TDS Deduction of Tax at lower rate or non-deduction of Tax

Tax Deduction at Source (TDS) on transfer of certain immovable properties (other than agricultural land) [Section 194LAA] [W.e.f. 1.10.2012] Every transfree, at the time of making payment or crediting any sum by way of consideration for transfer of immovable property (other than agricultural land), shall deduct tax, at the rate of 1% of such sum, if the consideration paid or payable for the transfer of such property exceeds (a) ` 50,00,000 in case such property is situated in a specified area; or (b) ` 20,00,000 in case such property is situated in any other area. Further, where the consideration paid or payable for the transfer of such property is less than the value adopted or assessed or assessable by any authority of a State Government for the purposes of payment of stamp duty, the value so adopted or assessed or assessable shall be deemed as consideration paid or payable for the transfer of such immovable property. Note : Specified Area :- Mumbai, Delhi, Kolkata, Chennai, Hyderabad, Bangaluru, Ahmedabad, Faridabad, Gurgaon, Gautam Budh Nagar, Ghaziabad, Gandhinagar and Secunderabad.

KSP Classes; PH : 9811880881, 9868308387

[ Amendment Page - 17]

CA. Krishna Shankar Prasad


Payer not to be deemed to be an assessee in default [Proviso to section 201(1)] [W.e.f. 1-7- 2012] In order to provide clarity regarding discharge of tax liability by the resident payee on payment of any sum received by him without deduction of tax, a new proviso has been inserted in section 201(1) to provide that the payer who fails to deduct the whole or any part of the tax on the payment made to a resident payee shall not be deemed to be an assessee in default in respect of such tax if such resident payee (i) has furnished his return of income under section 139; (ii) has taken into account such sum for computing income in such return of income; and (iii) has paid the tax due one the income declared by him in such return of income, and the payer furnishes a certificate to this effect from a chartered accountant in such form as may be prescribed. The date of payment of taxes by the resident payee shall be deemed to be the date on which return has been furnished by the payer.

Advance Payment of Tax


F

Exemption for Senior Citizens from payment of Advance Tax Section 207(2) :The provisions of advance tax shall not apply to an individual resident in India, who (i) does not have any income chargeable under the head Profits and gains of business or profession; and (ii) is of the age of 60 years or more at any time during the previous year.

Liability to pay advance tax in case of non-deduction of tax Section 209 [W.e.f. F.Y. 2012-13]
Where a person has received any income without deduction or collection of tax, he shall be liable to pay advance tax in respect of such income.

KSP Classes; PH : 9811880881, 9868308387

[ Amendment Page - 18]

CA. Krishna Shankar Prasad

REVISION TEST PAPER


CA-IPCC [For May, 2013 Exam]
P.1 Punjab and Power Ltd. engaged in the business of generation of power, furnishes the following particulars pertaining to P.Y. 2012-13 Compute the depreciation allowable under section 32 for A.Y. 2013-14, while computing its income under the head Profits and gains of business or profession. The company has opted for the deperciation allowance on the basis of written down value. Particulars 1. 2. 3. 4. 5. 6. 7. Opening Written down value of Plant and Machinery (15% block) as on 01.04.2012 (Purchase value ` 18,00,000) Purchase of second hand machinery (15% block) on 22.12.2012 for business purpose. Machinery Y (15% block) purchased and installed on 10.07.2012 for the purpose of power generation Acquired and installed for use a new air pollution control equipment on 31.07.2012 (100% block) New air conditioner purchased and installed in office premises on 08.09.2012 New machinery Z (15% block) acquired and installed on 23.12.2012 for the purpose of generation of power Sale value of an old machinery X, sold during the year (Purchase value ` 4,80,000, WDV as on 01.04.2012 ` 3,46,800) ` 5,78,000 2,00,000 8,00,000 8,50,000 3,00,000 3,25,000 3,10,000

KSP Classes; PH : 9811880881, 9868308387

[ Amendment Page - 19]

CA. Krishna Shankar Prasad


Ans:Computation of depreciation allowance under section 32 for the A.Y. 2013-14 Particulars Opening WDV as on 01.04.2012 Add : Plant and Machinery acquired during the year Second hand machinery Machinery Y Air conditioner for office Machinery Z Air pollution control equipment Less : Asset sold during the year Written down value before charging depreciation Normal depreciation 100% on air pollution control equipment Depreciation on plant and machinery put to use for less than 180 days @ 7.5% (i.e., 50% of 15%) Second hand machinery (` 2,00,000 7.5%) 15,000 Machinery Z (` 3,25,000 7.5%) 24,375 39,375 2,05,200 15% on the balance WDV being put to use for more than 180 days (` 13,68,000 15%) Additional depreciation Machinery Y (` 8,00,000 20%) Machinery Z (` 3,25,000 10%) Total depreciation 1,60,000 32,500 1,92,500 4,37,075 Nil 8,50,000 8,50,000 2,00,000 8,00,000 3,00,000 3,25,000 16,25,000 22,03,000 3,10,000 18,93,000 8,50,000 8,50,000 Nil 8,50,000 Plant and Machinery (15%) (` ) 5,78,000 Plant and Machinery (100%) (` )

Notes: (1) Power generation equipments qualify for claiming additional depreciation in respect of new plant and machinery from A.Y. 2013-14 (2) Additional depreciation is not allowed in respect of second hand machinery. (3) No additional depreciation is allowed in respect of office appliances. Hence, on depreciation is allowed in respect of air conditioner installed in office premises. (4) Additional depreciation is not allowed in respect of an asset whose actual cost is allowed as deduction in computing the income chargeable under the head Profit and Gains of business or profession. It is presumed that the new air pollution control equipment installed is eligible for 100% depreciation. Therefore, no additional depreciation is allowed in respect of the same.
KSP Classes; PH : 9811880881, 9868308387 [ Amendment Page - 20]

CA. Krishna Shankar Prasad


P.2 Mr. Sohan commenced operations of the businesses of setting up a warehousing facility for storage of food grains, sugar and edible oil on 1.4.2012. He incurred capital expenditure of ` 80 lakh, ` 60 lakh and ` 90 lakh, respectively, on purchased of land and building during the period January, 2012 to March, 2012 exclusively for the above businesses, and capitalized the same in its books of account as on 1st April, 2012. The cost of land included in the above figures are ` 50 lakh, ` 40 lakh and ` 70 lakh, respectively. Further, during the P.Y. 2012-13, it incurred capital expenditure of ` 20 lakh, ` 15 lakh & ` 10 lakh, respectively, for extension/ reconstruction of the building purchased and used exclusively for the above businesses. Compute the income under the dead Profits and gains of business or profession for the A.Y. 201314 and the loss to be carried forward, assuming that Mr. Sohan has fulfilled all the conditions specified for claim of deduction under section 35AD and has not claimed any deduction under Chapter VI-A. The profits from the business of setting up a warehousing facility (before claiming deduction under section 35AD and section 32) for the A.Y. 2013-14 is ` 16 lakhs, ` 14 lakhs and ` 55 lakhs, respectively

Ans:Computation of profits and gains of business or profession for A.Y. 2013-14 Particulars Profits from business of setting up of warehouse for storage of edible oil (before providing for depreciation under section 32) Less: Depreciation under section 32 10% of ` 30 lakh, being (` 90 lakh - ` 70 lakh + ` 10 lakh) Income chargeable under Profits and gains from business or profession ` (in Lakh) 55

3 52

KSP Classes; PH : 9811880881, 9868308387

[ Amendment Page - 21]

CA. Krishna Shankar Prasad


Computation of income/loss from specified business under section 35AD Food Sugar Total Grains ` (in Lakhs) (A) Profit from the specified business of setting up a warehousing 16 14 30 facility (before providing deduction under section 35AD) Particulars Less: Deduction under section 35AD (B) Capital expenditure incurred prior to 1.4.2012 (i.e., prior to commencement of business) and capitalized in the books of account as on 1.4.2012 (Excluding the expenditure incurred on acqusition of land) = ` 30 lakh (` 80 lakh - ` 50 lakh) and ` 20 lakh (` 60 lakh - ` 40 lakh) (C) Capital expenditure incurred during the P.Y.2012-13 (D) Total capital expenditure (B + C) (E) Deduction under section 35AD 150% of capital expenditure (food grains) 100% of capital expenditure (sugar) Total deduction u/s 35AD for A.Y. 2013-14 (F) Loss from the specified business of setting up and operating a warehousing facility (after providing for deduction under section 35AD) to be carried forward as per section 73A 75 (59) 75 35 35 110

30

20

50

20 50

15 35

35 85

(21) (80)

KSP Classes; PH : 9811880881, 9868308387

[ Amendment Page - 22]

CA. Krishna Shankar Prasad


P.3 Mr. Suresh sold his residential property on 2nd February, 2013 for ` 80 Lakh and paid brokerage@1% of sale price. He had purchased the said property in May 2002 for ` 18,00,000. In June, 2013, he invested ` 75 lakh in equity of M (P) Ltd. a newly incorporated SME manufacturing company, which constituted 63% of share capital of the said company. M (P) Ltd. utilized the said sum for the following purposes(a) Purchase of new plant and machinery durung July 2013 - ` 65 lakh (b) Included in (a) above are ` 6 lakh for purchase of computers and ` 8 lakh for purchase of cars (c) Air-conditioners purchased for ` 1 lakh, included in the (a) above, were installed at the residence of Mr. Suresh. (d) Amount deposited in specified bank on 28.09.2013 - ` 10 lakh Compute the chargeable capital gain for the A.Y. 2013-14. Assume that Mr. Suresh is liable to file his return of income on or before 30th September, 2013 and he files his return on 25.09.2013 Ans: Computation of taxable capital gains for A.Y. 2013-14 Particulars Gross consideration Less: Expenses on transfer (1% of the gross consideration) Net consideration Less: Indexed cost of acquisition
852 ) (` 18,00,000 447

` 80,00,000 80,0000 79,20,000 34,30,872 44,89,128

Less: Expenses under section 54GB ( 44,89,128 ` 60,00,0000) 79,20,000 34,00,855 10,88,273

Taxable capital gains

Deemed cost of new plant and machinery for exemption under section 54GB (1) Particulars Purchase cost of new plant and machinery acquired in july, 2013 Less: Cost of office appliances, i.e., computers Cost of vehicles i.e., cars Cost of air-conditioners installed at the residence of Mr. Suresh (2) Amount deposited in the specified bank before the due date of filing of return ` 6,00,000 8,00,000 1,00,000 15,00,000 50,00,000 10,00,000 60,00,000 ` 65,00,000

Deemed cost of new plant and machinery for exemption under section 54GB
KSP Classes; PH : 9811880881, 9868308387

[ Amendment Page - 23]

CA. Krishna Shankar Prasad


P.4 Mr. Rohan, aged 35 years, paid medical insurance premium of ` 12, 000 during the P.Y. 2012-13 to insure the health as well as the health of his spouse. He also paid medical insurance premium of ` 17,000 during the year to insure the health of his father, aged 64 years, who is not dependent on him. He contributed ` 2,400 to Central Government Health Scheme during the year. He has incurred ` 3,500 in cash on preventive health check-up of himself and his spouse and ` 4,500 by cheque on preventive health check-up of his father. Computer the deduction allowable under section 80D for the A.Y. 2013-14. Deduction allowable under section 80D for the A.Y.2013-14 Particulars ` ` Actual Maximum Payment deduction allowable

Ans:

A.

Premium paid and medical expenditure incurred for self and spouse 12,000 2,400 3,500 17,400 12,000 2,400 600 15,000

(i) Medical insurance premium paid for self and spouse (ii) Contribution to CGHS (iii) Exp. on preventive health check-up of self & spouse B. Premium paid and medical expenditure incurred for father, who is a senior citizen

(i) Mediclaim premium paid for father, who is over 60 years of age (ii) Expenditure on preventive health check-up of father

17,000 4,500 21,000

17,000 3,000 20,000 35,000

Total deduction under section 80D (15,000+20,000) Note:

(1) The total deduction under A. (i), (ii) and (iii) above should not exceed ` 15,000 Therefore, the expenditure on preventive health check-up for self and spouse would be restricted to ` 600, being (` 15,000 ` 12,000 ` 2400). (2) The total deduction under B. (i) and (ii) above should not exceed ` 20,000. Therefore, the expenditure on preventive health check-up for father would be restricted to ` 3,000, being (` 20,000 - ` 17,000) (3) In this case , the total deduction allowed on account of expenditure on preventive health checkup of self, spouse and father is ` 3,600 (i.e., ` 600 + ` 3,000), which is less than the maximum permissible limit of ` 5,000.

KSP Classes; PH : 9811880881, 9868308387

[ Amendment Page - 24]

Most Important Announcements


IPCC (May 2013 Examination) Paper - 4 (Taxation)
The following topics pertaining to SERVICE TAX law will not be included in the syllabus of IPCC (Taxation) :
(i) Section 66C - Determination of place of provision of service (ii) Place of provision of Service Rules, 2012 (iii) Section 66E - Declared services (iv) Section 68(2) and Reverse Charge notification (v) Export of Services (Rule 6A of the Service Tax Rules, 1994) (vi) Mega Exemption (N.No. 25/2012 Service Tax) (vii) Abatement [Notification No. 26/2012 S.T.]

Chapter - 1

Basic Concept of Service Tax


[Finance Act 1994]
) Extent, Commencement and Application Section 64

:-

This Chapter extends to the whole of India except the State of Jammu and Kashmir. It shall apply to taxable services provided.

Service Tax is Consumption - Based Destination Tax

It means that service tax is leviable only if the services are rendered by service provider i.e. consumed by service receiver at a destination, where Finance Act, 1994 is applicable.

) Definition Section 65B


Actionable Claim [Section 65B(1)] Actionable Claim shall have the meaning assigned to it in Section 3 of the Transfer of Property Act, 1882. As Per Section 3 of the Transfer of Property Act, 1882 Actionable claim means a claim to (a) any debt, other than a debt secured by mortgage of immovable property or by hypothecation or pledge of movable property; or (b) any beneficial interest in movable property not in the possession, either actual or contructive, of the claimant, which the Civil Courts recognize as affording grounds for relief; whether such debt or beneficial interest be existent, accruing, conditional or contingent. Business Entity [Section 65B(17)] Business entity means any person ordinarily carrying out any activity relating to industry, commerce or any other business or profession. Declared Services [Section 65B(22)] Declared service means any activity carried out by a person for another person for consideration and declared as such under section 66E. Goods [Section 65B(25)] Goods means every kind of movable property other than actionable claim and money; and includes securities, growing crops, grass, and things attached to or forming part of the land which are agreed to be severed before sale or under the contract of sale.

CA. Krishna Shankar Prasad, (9868308387)

Basic Concept of Service Tax [2]

India [Section 65B(27)] India means (a) the territory of the Union. (b) its territorial waters, contintal shelf, exclusive economic zone or any other maritime zone as defined in the Territorial Waters, Contintal Shelf, Exclusive Economic Zone and other Maritime Zones Act, 1976. (c) the seabed and the subsoil underlying the territorial waters. (d) the air space above its territory waters; and (e) the installations, structures and vessels located in the continental shelf of India and the exclusive economic zone of India, for the purposes of prospecting or extraction or production of mineral oil and natural gas and supply thereof. Non Taxable Territory [Section 65B(35)] Non-taxable territory means the territory which is outside the taxable territory. Person [Section 65B(37)] Person includes, (a) an Individual, (b) a Hindu undivided family, (c) a company, (d) a society, (e) a limited liability partnership, (f) a firm, (g) an association of persons or body of individuals, whether incorporated or not, (h) Government, (i) a local authority, or (j) every artificial juridicial person, not falling within any of the preceding sub-clauses.

CA. Krishna Shankar Prasad, (9868308387)

Basic Concept of Service Tax [3]

Service [Section 65B(44)] Service means any activity carried out by a person for another for consideration, and includes a declared service, but shall not include (a) An activity which constitutes merely a transfer of title in goods or immovable property, by way of sale, gift or in any other matter. (b) An activity which constitutes merely "deemed sale" of goods or immovable property as given in clause (29A) of article 366 of the Constitution. (c) An activity which constitutes merely a transaction in money or an actionable claim. (d) Provision of service by an employee to the employer in the course of, or in relation to, his employment. (e) Fees taken in any Court or Tribunal established under any law for the time being in force. (f) Functions performed by the Members of Parliament, Members of State Legislative, Members of Panchayats, Members of Municipalities and Members of other local authorities who receive any consideration in performing the functions of that office as such members. (g) Duties performed by any person who holds any post in pursuance of the provisions of the Constitution in that capacity. (h) Duties performed by any person as a Chairperson or a Member or a Director in a body established by the Central Government or State Governments or local authority and who is not deemed as an employee before the commencement of this section. Taxable Services [Section 65B(51)] Taxable service means any service on which service tax is leviable under section 66B. Taxable Territory [Section 65B(52)] Taxable Territory means the territory to which the provisions of this Chapter apply.

) Charge of Service Tax Section 66B


Charge of service tax on and after Finance Act, 2012 There shall be levied a tax at the rate of 12% on the value of all services, other than those services specified in the negative list, provided or agreed to be provided in the taxable territory by one person to another and collected in such manner as may be prescribed. [Service tax is to be increased by Education cess @2% and Secondary & Higher education cess @1%. Hence, effective rate of charge of service tax is 12.36% of value of taxable service.

CA. Krishna Shankar Prasad, (9868308387)

Basic Concept of Service Tax [4]

) Principles of interpretation of Specified descriptions of Services of Bundled Services Section 66F


(a) Reference to main service not to include incidental service : As per Section 66F(1), unless otherwise specified, reference to a main service shall not include reference to a service which is used for providing main service. (b) Specific description shall be preferred over a general description : As per Section 66F(2), where a service is capable of differential treatment for any purpose based on its description, the most specific description shall be preferred over a more general description. For instance, differential treatment can arise as for some of the services where tax is payable by service recipient and not the service provider. (c) Natural bundle of Service : Classified as service which gives it the essential character: If various elements of such service are naturally bundled in the ordinary course of business, it shall be treated as provision of the single service which gives such bundle its essential character. (d) Artificial bundle of Service : Classified as one which attracts the highest tax liability: If various elements of such service are not naturally bundled in the ordinary course of business, it shall be treated as provision of the single service which results in highest liability of service tax. Bundled Service : Bundled Service means a bundle of provision of various services wherein an element of provision of one service is combined with an element or elements of provision of any other service or services.

Basic Concept of Service Tax (P & S) 5

Problem and Solutions


P.1 What are the sources of Service Tax Law? P.2 Which Act and Rule governs the levy of Service Tax in India? P.3 State with reasons in brief whether the following statements are correct or incorrect with reference to the provisions of service tax Service tax provisions are not applicable in J & K because the Central Government concurrence was not obtained in respect of Finanace Act, 1944. P.4 Is an unincorporated association liable to pay any service tax? P.5 Explain as to how and when the amendments made in the finance bill in respect of the service tax matter come into the force. P.6 Briefly explain the nature of the service tax. P.7 Briefly explain the charge of the service tax. P.8 What are Different approaches to service tax.

Basic Concept of Service Tax (P & S) 6

[Solutions]
P.1 Sources of Service Tax Law :(i) Chapter V of Finance Act 1994. (ii) Rules on Service Tax: Service Tax Rules, 1994 Service Tax (Determination of Value) Rules, 2006 Cenvat Credit Rules, 2004. Place of Provision of Services Rules, 2012 Service Tax (Registration of Special Category of Persons) Rules, 2005 Service Tax (Advance Rulings) Rules, 2003 Point of Taxation Rules, 2011. Service Tax (Publication of Names) Rules, 2008 Service Tax (Settlement of Cases) Rules, 2012 Service Tax (Compounding of Offences) Rules, 2012 (iii) Circulars of Central Board of Excise & Customs. (iv) Judicial Decisions. P.2 Chapter V of the Finance Act, 1994 & Service Tax Rules, 1994. P.3 Solution :Incorrect. It is not Finance Act, 1944 but the Finance Act, 1994. P.4 Yes, An unincorporated association is liable to pay service tax. Unincorporated associations or body of persons providing taxable services to its members for cash, deferred payment or any other valuable consideration are liable to pay Service Tax. P.5 Service tax amendments made by a Finance Bill are generally applicable from the date when the Finance Bill is given assent by the President. Immediately after the Presidents assent, the Finance Bill becomes Finance Act. P.6 Service tax is in nature of Indirect Tax. Service Tax is a tax on service. If there is no service, there is no tax. There is no Service Tax Act as such. Service tax is imposed by ameding Chapter V of Finance Act, 1994 from time to time. P.7 Section 66B of Finance Act, 1994.

Basic Concept of Service Tax (P & S) 7

P.8 There are two approaches to taxation of services (a) Selective approach :- In the case of selective approach, only selective services are subject to service tax. In this case, the legislator attempts to specify and list the services that would be taxable and the scope of coverage of each service. (b) Comprehensive approach :- In the case of comprehensive approach all services are taxable and a negative list is specified for services, which are not taxable. w.e.f. 1-7-2012, India has shifted from selective approach to comprehensive approach.

Chapter - 2

Valuation of Taxable Services


F Valuation of Taxable Services for Charging Service Tax Section 67
Valuation of taxable services for charging service tax (1) Subject to the provisions of this chapter, service tax chargeable on any taxable service with refeence to its value shall,Case (i) In a case where the provision of service is for a consideration in money In a case where the provision of service is for a consideration not wholly or partly consisting of money Value of Service The gross amount charged by the service provider for such service provided or to be provided by him; Such amount in money, with the addition of service tax charged, is equivalent to the consideration; The amount as may be determined in the prescribed manner.

(ii)

(iii) In a case where the provision of service is for a consideration which is not ascertainable

(2) Where the gross amount charged by a service provider, for the service provided or to be provided is inclusive of service tax payable, the value of such taxable service shall be such amount as, with the addition of tax payable, is equal to the gross amount charged. (3) The gross amount charged for the taxable service shall include any amount received towards the taxable service before, during or after provision of such service. (4) Subject to the provisions of sub-sections (1), (2) and (3), the value shall be determined in such manner as may be prescribed. Key Notes :(i) Money [Section 65B(33)] :- Money means legal tender, cheque, promissory note, bill of exchange, letter of credit, draft, pay order, traveller cheque, money order, postal or electronic remittance or any such similar instrument but shall not include any currency that is held for its numismatic value.

(ii) Meaning of Gross amount charged :- Gross amount charged includes payment by cheque, credit card, deduction from account and any form of payment by issue of credit notes or debit notes and book adjustment.

CA. Krishna Shankar Prasad, (9868308387)

Valuation of Taxable Services [9]

F Rule 3 of Service Tax (Determination of Value) Rules, 2006 Manner of determination of value Subject to the provisions of section 67, the value of taxable service, where such value is not ascertainable, shall be determined by the service provider in the following manner:(a) the value of such taxable service shall be equivalent to the gross amount charged by the service provider to provide similar service to any other person in the ordinary course of trade and the gross amount charged is the sole consideration; (b) where the value cannot be determined in accordance with clause (a), the service provider shall determine the equivalent money value of such consideration which shall, in no case be less than the cost of provision of such taxable service.

F Date of Determination of Rate of Tax, Value of Taxable Service and Rate of Exchange Section 67A
The Rate of service tax, value of a taxable service and rate of exchange, if any, shall be the rate of service tax or value of a taxable service or rate of exchange, as the case may be, in force or as applicable at the time when the taxable service has been provided or agreed to be provided.

In case of Free Services :No Service tax is leviable if free services are provided by the provider.

CBDT Circular : 4/2008 (Date 28-04-2008) TDS under section 194 I (Rental Amount):Amount of Rent Paid/Payable without Including the Service Tax. TDS under section 194J (Professional or technical Services):Any sum paid as professional and technical fees. (Including Service tax)

Valuation of Taxable Services (P & S) 10

Problem and Solutions


P.1 Should service tax be paid even if not collected from the client or Service Receiver? P.2 Will the payment to a hotilier of ` 20,000 on behalf of an architect by a service receiver be included in the value of taxable services? P.3 Can it be said that if the taxable service is not capable of ascertainment, the same cannot form part of value of taxable services? P.4 Mr. Suresh a service provider who pays service tax regularly, was of the opinion that a particular service was not liable for service tax. He, therfore, did not charge service tax in his bill. He received the bill amount without service tax. How will service tax liability of Mr. Suresh be determined in such case? P.5 Ms Rani rendered taxable services to a client. A bill of ` 5,00,000 was raised on 20/4/2012. ` 3,00,000 was received from a client on 30/5/2012 and the balance on 13/6/2012. No service tax was separately charged in the bill. The questions are: (a) Is Ms Rani liable to pay service tax, even though the same has not been charged by her? (b) In case she is liable, what is the value of taxable services and the service tax payable? P.6 Ms. Dipti, a proprietress of Royal Security Agency received ` 2,00,000 by an account payee cheque, as advance while signing a contract for providing taxable services. She receives ` 4,00,000 by credit card while providing the service and another ` 7,00,000 by a pay order after completion of service on 28/2/2013. All three transactions took place during financial year 2012-2013. She seeks your advice about her liability towards value of taxable service and the service tax payable by her. (a) If Ms Dipti is a new service provider (b) If Ms Dipti is a not a new service provider {service tax was levied last year). P.7 Mr. Shyam has rendered freely, a service to a client which is taxable, but has not charged or received any fee from a client. Is service tax payable on such free services? P.8 Whether free services after the sale of motor vehicle given by the authorized dealer for which they are reimbursed by the vehicle manufacturer are subject to the service tax? P.9 How the value of taxable services determind when the consideration against taxable services is received in other than monetary terms? P.10 M/s. Subhay Coaching Ltd. provides coaching in the field of engineering. One of the students, Mr. X, paid Rs. 15,000 in cash and a gold chain valuing Rs. 35,000 (cost to Mr. X's father who is a jeweller. Rs. 28,000) to M/s. PQR Ltd. Compute the amount of service tax payable by M/s. Subhay Ltd. thereon @ 12.36%. Make suitable assumptions. P.11 State with reason in brief whether the following statement is TRUE or FALSE with reference to the provisions of Service Tax: [April 2012] Mr. Arun, an architect has received the fees of ` 6,30,000 after the deduction of Income Tax of ` 70,000. The Service Tax is payable on ` 6,30,000.

Valuation of Taxable Services (P & S) 11

[Solutions]
P.1 Yes It has to be paid even if the service provider fials to charge the service tax from the service receiver. If however, the service provider fails to recover service tax from the client in lieu of services rendered, it will be taken to be inclusive of service tax. Value of taxable service = Gross amount charged 100 (100 + Rate of tax) Service tax = Gross amount charged Rate of tax (100 + Rate of tax) P.2 Yes. Section 67 of Finance Act, 1994 provides that taxable value shall be gross amount charged for the service. Service Tax (Determination of Value) Rule, 2006 provides that where any expenditure or costs are incurred by the service provider in the course of providing taxable service, all such expnditure or costs shall be treated as consideration for the taxable service provided or to be provided and shall be included in the value for the purpose of charging service tax on the said service. P.3 Section 67 of Finance Act, 1994 provides that in a case where the provision of service is for a consideration which is not ascertainable, then value of service shall be determined in such manner as prescribed by Service Tax (Determination of Value) Rules, 2006. P.4 Mr. Suresh is liable to pay service tax even though the Service Tax has not been collected by his. It has to be paid even if the service provider fials to charge the service tax from the service receiver. If however, the service provider fails to recover service tax from the client in lieu of services rendered, it will be taken to be inclusive of service tax. P.5 (a) Rani is liable to pay service tax even though the service tax has not been collected by her. It has to be paid even if the service provider fials to charge the service tax from the service receiver. If however, the service provider fails to recover service tax from the client in lieu of services rendered, it will be taken to be inclusive of service tax.

Valuation of Taxable Services (P & S) 12

(b)

Gross amount charged 100 Value of taxable service = (100 + Rate of tax) = 5,00,000 100 (100 + 12.36) Gross amount charged Rate of tax Service Tax = = (100 + Rate of tax) 5,00,000 12.36 (100 + 12.36) = 55,002 (Due Date = 5 July, 2012) = 4,44,998

P.6 (a) If Dipti is a new service provider, then there is no service tax liability so long as the value of taxable services does not exceed ` 10,00,000. However, registration is required if the value of taxable of services exceeds ` 9,00,000. Out of ` 13,00,000, ` 10,00,000 is not chargeable to service tax. If service tax is not separately charged, it will be assumed that ` 3,00,000 is inclusive of service tax. Gross amount charged 100 Value of taxable service = (100 + Rate of tax) 3,00,000 100 = (100 + 12.36) Service Tax = Gross amount charged Rate of tax = (100 + Rate of tax) (100 + 12.36) 3,00,000 12.36 = 33,001 = 2,66,999

(b) If Dipti is not a new service provider (service tax was levied last year), then the entire amount of ` 13,00,000 will be subject to service tax. If service tax is not separately charged, it will be assumed that ` 13,00,000 is inclusive of service tax.

Value of taxable service =

13,00,000 100 = 11,56,995 (100 + 12.36)

Service Tax =

13,00,000 12.36 = 1,43,005 (100 + 12.36)

P.7 Mr. Shyam shall not be liable to pay service tax on services rendered without any consideration. P.8 Free after sale service is provided by authorized dealer to the purchaser of motor vehicles. The vehicle manufacturer later on reimburses the amount. Since for the service provider, it is not free service, it is liable for service tax.

Valuation of Taxable Services (P & S) 13

P.9 Rule 3 of Service Tax (Determination of Value) Rules, 2006. P.10 Solution :It is a case covered by section 67(1)(ii), whereunder it is provided that in a case where the provision of service is for a consideration not wholly or partly consisting of money, the value shall be such amount in money as, with the addition of service tax charged, is equivalent to the consideration. In this case, M/s. Subhay Coaching Ltd. has received ` 15,000 in cash and gold valuing ` 35,000, therefore, the total consideration received by M/s. Subhay Coaching Ltd. is ` 50,000 (inclusive of taxes). Hence, the service tax payable by M/s. Subhay Coaching Ltd. = ` 50,000 12.36% 112.36% = ` 5,500 approx. P.11 Statement is False: Service tax is payable on gross amount charged by the service provider for the services provided or to be provided. Service Tax is payable On ` 7,00,000. If service tax is not separately charged, it will be assumed that ` 7,00,000 is inclusive of service tax. Gross amount charged Rate of tax (100 + Rate of tax) 7,00,000 12.36 (100 + 12.36) = ` 77,002

Chapter - 3

Payment of Service Tax and Due Dates


F Payment of Service Tax Section 68
Payment of Service tax (1) Every person providing taxable service to any person shall pay service tax at the rate specified in section 66B in such manner and within such period as may be prescribed. (2) Notwithstanding anything contained in sub-section (1), in respect of such taxable services as may be notified by the Central Government in the Official Gazette, the service tax thereon shall be paid by such person and in such manner as may be prescribed at the rate specified in section 66B and all the provisions of this Chapter shall apply to such person as if he is the person liable for paying the service tax in relation to such service. Provided that the Central Government may notify the service and the extent of service tax which shall be payable by such person and the provisions of this Chapter shall apply to such person to the extent so specified and the remaining part of the service tax shall be paid by the service provider.

F Rule 4A of Service Tax Rules, 1994 (Amendment w.e.f. 1-4-2012)


Service provider to issue bill : Every person providing any taxable service shall issue an invoice, bill, challan signed by such person or person authorised by him in respect of such taxable service provided or agreed to be provided. The invoice, bill, challan shall be issued not later than 30 days from (a) the date of completion of such taxable services, or (b) the receipt of payment towards the value of such taxable service whichever is earlier. Note : (1) Where the provider of taxable service is a banking company or a financial insititution including a non-banking financial company, or any other person, providing service to any person, in relation to banking and other financial services, the period within which the invoice, bill or challan, as the case may be is to be issued, shall be 45 days. (2) Point of Taxation of amount upto ` 1000 received in excess of invoice amount :wherever the provider of taxable service receives a payment upto ` 1000 in excess of the amount indicated in the invoice, the point of taxation to the extent of such excess amount, at the option of the provider of taxable service, shall be determined on the basis of invoice issued instead of date of receipt of payment. Where such option is exercised, no invoice is required to be issued to such extent.

CA. Krishna Shankar Prasad, (9868308387)

Payment of Service Tax & Due Dates [15]

Rule 5 of Service Tax Rules, 1994 :- Maintenance of Records


(1) (2) Records maintained under by any other law- acceptable as Service Tax Records Every assessee shall furnish to the Superintendent of Central Excise at the time of filing of return for the first time or the 31st day of January, 2008, whichever is later, a list in duplicate of (i) all the records prepared or maintained by the assessee for accounting of transactions in regard to (a) providing of any service (b) receipt or procurement of input services and payment for such input services; (c) receipt, purchase, manufacture, storage, sale, or delivery, as the case may be, in regard to inputs and capital goods; (d) other activities, such as manufacture and sale of goods, if any. (ii) all other financial records maintained by him in the normal course of business. (3) (4) Period of preservation - Minimum 5 Years. Records must be made available for inspection and examination by Central Excise Officer.

Rule 5A of Service Tax Rules, 1994 :-Access to a registered premises (1) An officer authorised by the Commissioner in this behalf shall have access to any premises registered under these rules for the purpose of carrying out any scrutiny, verification and checks as may be necessary to safeguard the interest of revenue. Every assessee shall, on demand, make available to the officere authorised under sub-rule (1) or the audit party deputed by the Commissioner or the Comptroller and Auditor General of India, within a reasonable time not exceeding fifteen working days from the day when such demand is made, or such further period as may be allowed by such officer or the audit party, as the case may be (i) the records as mentioned in rule 5(2), (ii) trial balance or its equivalent, and (iii) the income-tax audit report, if any, under section 44AB of the Income-tax Act, 1961 for the scrutiny of the officer or audit party, as the case may be.

(2)

CA. Krishna Shankar Prasad, (9868308387)

Payment of Service Tax & Due Dates [16]

Point of Taxation Rules, 2011


F Determination of Taxable event Point of Taxation of Taxable Services
Rule 2(c) :- Continuous Supply of Service
Continuous supply of service means any service which is provided, or agreed to be provided continously or on recurrent basis, under a contract, for a period exceeding three months with the obligation for payment periodically or from time to time, or where the central government, by a notification in the official gazette, prescribes of a particular service to be a continuous supply of service, whether or not subject to any condition.

Rule 2A Date of payment


For the purposes of these rules, date of payment shall be(a) the date on which the payment is entered in the books of accounts or (b) the date on which the payment is credited to the bank account of the person liable to pay tax, whichever is earlier. However, as per proviso to Rule 2A,(A) the date of payment shall be the date of credit in the bank account when (i) there is a change in effective rate of tax or when a service is taxed for the first time during the period between such entry in books of accounts and its credit in the bank account; (ii) the credit in the bank account is after four working days form the date when there is change in effective rate of tax or a service is taxed for the first time; and (iii) the payment is made by way of an instrument which is credited to a bank account, (B) if any rule requires determination of the time or date of payment received, the expression date of payment shall be construed to mean such date on which the payment is received.

CA. Krishna Shankar Prasad, (9868308387)

Payment of Service Tax & Due Dates [17]

Rule 3 :- (Amendement w.e.f. 1-4-2012)


Point of Taxation : Thus, point of taxation i.e. deemed date of provision of service shall be (a) the date of issue of invoice, if such invoice is issued within 30 / 45 days (as the case may be) of completion of service [in case of continuous supply of service, the invoice must be issued within 30 days from each of the event dates specified in the contract for payment by service receiver]. (b) the date of completion of service, if such invoice is not issued within 30 / 45 days (as the case may be) as aforesaid, (c) the date on which the payment is received by the service provider, whichever is the earliest.

Rule 4 :Determination of point of taxation in case of change in effective rate of tax] (a) In case a taxable service has been provided before the change in effective rate of tax Invoice Issued (i) After the Change (ii) Before the Change (iii) After the Change Payment Received After the Change After the Change Before the Change Point of Taxation whichever is earlier Date of issuing of Invoice Date of Payment

(b) In case a taxable service has been provided after the change in effective rate of tax Invoice Issued (i) Before the Change (ii) Before the Change (iii) After the Change Payment Received After the Change Before the Change Before the Change Point of Taxation Date of Payment whichever is earlier Date of issuing of Invoice

CA. Krishna Shankar Prasad, (9868308387)

Payment of Service Tax & Due Dates [18]

Rule 5. Payment of tax in cases of new services - Where a service is taxed for the first time then, (a) no tax shall be payable to the extent the invoice has been issued and the payment received against such invoice before such service became taxable : (b) no tax shall be payable if the payment has been received before the service becomes taxable and invoice has been issued within 14 days of the date when the service is taxed for the first time. Rule 6. In case of continuous supply of service - Omitted w.e.f. 1-4-2012 as it has been merged with rule 3. Rule 7. Point of Taxation in case of specified services Rule 7 is applicable if service tax is payable by the recipient of service under reverse charge mechanism under section 68(2). If payment is made by service recipient within 6 months of the date of invoice Point of taxation will be the date on which payment is made by the recipient of service to the service provider. If payment is not made by service recipient within 6 months of the date of invoice Rule 7 will not be applicable. Point of taxation will be decided by other rules. If no other rule is applicable, point of taxation will be determined by rule 3. When service is received from an associated enterprise located outside India If service is received from an associated enterprise located outside India, Point of taxation will be (i) date of debit in the books of account of the recipient of service; or (ii) date of payment by the recipient of service to the service provider, Rule 8. Determination of point of taxation in case of copyrights, etc. In respect of royalties and payments pertaining to copyrights, trademarks, designs or patents, where the whole amount is not ascertainable at the time of provision of service, service shall be deemed to have been provided each time payment in respect of such service is received or invoice is issued by service provider, whichever is earlier. Rule 8A. Determination of Point of Taxation in other cases Where the point of taxation cannot be determined as per these rules as the date of invoice or the date of payment or both are not available, the Central Excise Officer, may, require the concerned person to produce such accounts, documents or other evidence as he may deem necessary and after taking into account such material and the effective rate of tax prevalent at different points of time, shall, by an order in writing, after giving and opportunity of being heard, determine the point of taxation to the best of his judgement.

whichever is earlier.

CA. Krishna Shankar Prasad, (9868308387)

Payment of Service Tax & Due Dates [19]

Rule 6 of Service Tax Rules 1994 - Due Date for Payment of Service Tax :I. Individuals, Proprietary concerns and partnership firms Quarters Due dates 1st April 1st July 1st October 1st January II. to to to to 30th June 30th September 31st December 31st March 5th July 5th October 5th January 31st March Due dates (in case of E-payment) 6th July 6th October 6th January 31st March

Persons other than individuals, proprietary concerns and partnership firms Months Due dates Due dates (in case of E-payments) th April 5 May 6th May May 5th June 6th June June 5th July 6th July July 5th August 6th August August 5th September 6th September September 5th October 6th October October 5th November 6th November November 5th December 6th December December 5th January 6th January January 5th February 6th February February 5th March 6th March March 31st March 31st March

Note : In case of Individuals and partnership firms (including LLP) whose aggregate value of taxable services provided from one or more premises is ` 50 lakh or less in the previous financial year, the service provider shall have the option to pay tax on taxable services provided or agreed to be provided by him up to a total of ` 50 lakh in the curretn financial year, by the dates specified in this sub-rule with respect to the month or quarter, as the case may be, in which payment is received. (Amendment w.e.f. 1-4-2012)

CA. Krishna Shankar Prasad, (9868308387)

Payment of Service Tax & Due Dates [20]

Rule 6 of Service Tax Rules, 1994 Payment of Service Tax


(1) According to Section 68 of the Finance Act, 1994 read with Rule 6(1) of the Service Tax Rules 1994, the service tax shall be paid to the credit of the Central Government in the following manner. Assessee Individual, Proprietary firm, or, a Partnership firm Duration of payment Quarterly Time of Payment (1) If tax is paid electronically throught internet banking : by 6th of the month (2) In any other case : by the 5th of month, immediately following the quarter in which the service is deemed to be provided as per the rules framed in this regard. [w.e.f. 1-4-2011] Monthly (1) If tax is paid electronically through internet banking: by 6th of the month (2) In any other case : by the 5th of the month, immediately following the month in which the service is deemed to be provided as per the rules framed in this regard [w.e.f. 1-4-2011] Service deemed to be provided in month/quarter ending on 31st March - Payable by 31st March itself [w.e.f. 1-4-2011] : The service tax on the service deemed to be provided in the month of March, or the quarter ending in March, as the case may be, shall be paid to the credit of the Central Government by the 31st day of March of the calendar year. (1A) Without prejudice to the provisions contained in sub-rule (1), every person liable to pay service tax, may, on his own choice, pay an amount as service tax in advance, to the credit of the Central Government and adjust the amount so paid against the service tax which he is liable to pay for the subsequent period : Provided that the assessee shall (i) intimate the details of the amount of service tax paid in advance, to the jurisdictional Superintendent of Central Excise within a period of fifteen days from the date of such payment, and

Any other assessee

(ii) indicate the details of the advacne payment made, and its adjustment, if any in the subsequent return to be filed under section 70 of the Act.

CA. Krishna Shankar Prasad, (9868308387)

Payment of Service Tax & Due Dates [21]

(2)

Method of Deposit :The assessee shall deposit the service tax liable to be paid by him with the bank designated by CBEC for this purpose in form GAR-7 or in any other prescribed manner. Provided that an assessee shall deposit the service tax electronically through internet banking if he has paid the total service tax of ` 10 lakh or more (including the amount of service tax paid by utilisation of CENVAT credit) in the preceding financial year. Failure to pay service tax electronically when the person is so required shall attract a penalty of an amount upto ` 10,000.

(2A) If the assessee deposits the service tax by cheque :The date of presentation of cheque to the bank designated shall be deemed to be the date on which service tax has been paid subject to realization of that cheque.

(3) Adjustment of Excess Service Tax Paid :Where an assessee has issued an invoice, or received any payment, against a service to be provided which is not so provided by him either wholly or partly for any reason or where the amount of invoice is renegotiated due to deficient provision of service, or any terms contained in a contract, the assessee may take the credit of such excess service tax paid by him, if the assessee (a) has refunded the payment or part thereof, so received for the service provided to the person from whom it was received; or (b) has issued a credit note for the value of the service not so provided to the person to whom such an invoice had been issued. (4) Provisional Assessment :When an assessee is, for any reason, unable to correctly estimate, on the date of deposit, the actual amount payable for any particular month or quarter, as the case may be, he may make a request in writing to the AC/DC of Central Excise, giving reasons for payment of service tax on provisional basis. On receipt of such request, the AC/DC may allow payment of service tax on provisional basis on such value of taxable service as may be specified by him. (4A) & (4B) When excess amount of Service Tax is paid for any other reasons :Where an assessee has paid to the credit of Central Government any amount in excess of the amount required to be paid the assessee may adjust such excess against the service tax liability for the next quarter/month. Adjustment shall be possible only if the following conditions are satisfied :the excess amount paid is not on account of reasons involving interpretation of law, taxability, valuation or applicability of any exemption notification.

Payment of Service Tax & Due Dates (P & S) 22

Problem and Solutions


P.1 Who is liable to pay service tax in relation to service provided by a goods transport agency? P.2 Who is responsible to pay service tax when the receipient of sponsorship is located outside India? P.3 List the documents to be submitted along with the First Service Tax Return. P.4 Determination of Point of Taxation :Date of Completion of Services (i) (ii) (iii) (iv) May 15, 2012 May 15, 2012 May 15, 2012 May 20, 2012 Date of Invoice June 11, 2012 June 30, 2012 May 25, 2012 June 25, 2012 Date of which payment Rec. June 30, 2012 May 30, 2012 May 20, 2012 May 15, 2012 (Part) June 12, 2012

P.5 A service provider maintains books of accounts on mercantile basis relating to the taxable services provided by him, will service tax be payable on the accrual basis? P.6 Determine the point of taxation in the following cases given that : Rate of service tax upto 31-3-2012 is 10%; while that on or after 1-4-2012 is 12%. Compute the amount of service tax payable : Bill No. B-1 B-2 B-3 Value of Taxable Date of actual Date of issue Date of receipt of service (Rs.) provision of service of invoice payment 5,00,000 18,00,000 20,00,000 01-03-2012 31-03-2012 30-03-2012 11-03-2012 14-04-2012 12-04-2012 01-06-2012 22-06-2012 31-03-2012

Payment of Service Tax & Due Dates (P & S) 23

P.7 Certain services have become taxable for the first time on 1-6-2012. The Following Details are furnished by a Service provider in respect of the services for the financial year 2012-13. You are required to compute the amount of Service tax payable @ 12.36%.
Bill No. B-001 B-002 B-003 B-004 B-005 Value of Date of actual Date of issue Date of receipt service ( ` ) provision of service of invoice of payment 25,00,000 18,00,000 12,50,000 22,00,000 15,00,000 10-04-2012 06-05-2012 30-05-2012 05-06-2012 07-07-2012 22-04-2012 27-05-2012 12-06-2012 18-06-2012 10-07-2012 05-05-2012 30-05-2012 10-06-2012 30-05-2012 30-06-2012 01-06-2012 Amount exclusive of Taxes 25,00,000 14,00,000 4,00,000 12,50,000 22,00,000 15,00,000

P.8 A particular service has been brought into service tax net with effect from 1/9/2012. Mr. Santosh has provided this service on 10/8/2012, the payment for the same was received on 20/08/2012, and the invoice was issued on the 18/08/2012. Is the service tax payable on the same? P.9 What are the due dates for payment of Service Tax? P.10 Who is liable to make E-payment of Service Tax? P.11 Rajendra has paid the amount of service tax for the quarter ending September 30, 2012 by cheque. The date of presentation of cheque to the designated bank is October 5, 2012 and it is realised by the bank on October 7, 2012? What is the date of payment of service tax in this case? Whether any interest and penalty is attracted in this case? P.12 Ajay Ltd. has agreed to render services to Mr. Guru. The following are the chronological events: Conract for services entered into on 31.8.2012 Advance received in September, 2012 towards all services Total value of services, billed in February, 2013 Above includes non-taxable services of Balance amount is received in March, 2013 ` 60,000 2,10,000 70,000

When does the liable to pay service tax arise and for what amount ? Contract contains clear details of services; consideration and service tax are charged separately, as mutually agreed upon. Note : The value of services provided during the preceding year was ` 38 lakhs.

Payment of Service Tax & Due Dates (P & S) 24

P.13 Mr. Arun, a consulting engineer raised a bill of ` 2,24,720 (including service tax) on his client for consulting services rendered by him in the month Aug., 2012. A partial payment of ` 1,68,540 was received by Mr. Arun in the month March, 2013. Compute the service tax amount payable to Mr. Arun and the due date by which service tax can be deposited. Note : The value of taxable services rendered during the preceding year was ` 34 lakhs. P.14 An assessee who has collected Service Tax from a client is unable to perform the service. Briefly explain the situations in which and the conditions subject to which he can adjust the service tax relating to above, against his forthcoming service tax liability. P.15 Computation of Service Tax : [April 2012] ABC Limited provided taxable services, the gross amount charged in respect of which is ` 20,00,000 plus ` 2,47,200 towards service tax charged separately in the bill, the total bill amount being ` 22,47,200. Compute the amount of service tax payable by him, in the following cases -(i) the client pays ` 22,00,000 in full and final settlement of the bill; or (ii) the client pays ` 20,00,000 in full and final settlement of the bill; or (iii) the client pays ` 16,00,000 in full and final settlemetn of the bill. P.16 Is the service provider allowed to pay service tax on the provisional basis?

Payment of Service Tax & Due Dates (P & S) 25

[Solutions]
P.1 Rule 2(1)(d) of Service Tax Rules, 1994. P.2 In relation to sponsorhip services, the person sponsoring the event is liable to pay service tax if he is located in India. [Rule 2(1)(d) of Service Tax, Rule 1994] However, where the recipient of sponsorship services is located outside India, the service provider will be responsible to pay service tax. P.3 The following documents should be submitted along with First Service Tax return :(i) all the records prepared or maintained by the assessee for accounting of transactions in regard to (a) providing of any service, whether taxable or exempted; (b) receipt or procurement of input services and payment for such input services; (c) receipt, purchase, manufacture, storage, sale, or delivery, as the case may be, in regard to inputs and capital goods; (d) other activities, such as manufacture and sale of goods, if any. (ii) all other financial records maintained by him in the normal course of business. (iii) Copy of GAR-7 Challan. P.4 (i) June 11, 2012 Invoice issued in 30 days and before receipt of payment. (ii) May 15, 2012 Invoice not issued within 30 days and payment received after completion of service. (iii) May 20, 2012 Invoice issued in 30 days but payment received before invoice. (iv) May 15, 2012 (for partly payment rec.) May 20, 2012 Invoice not issued in 30 days, Part Payment before completion remaining later.

Payment of Service Tax & Due Dates (P & S) 26

P.5 Rule 3 of the Point of Taxation Rules, 2011. [Rule 6(1) of Service Tax Rules, 1994] P.6 Solution :The point of taxation under Rule 4 of the Point of Taxation Rules, 2011 and applicable rate of service tax under Rule 5B of the Service Tax Rules, 1994 along with computation of service tax is as follows Bill No. B-1 Value (Rs.) 5,00,000 Point of Taxation under Rule 4 of Point of Taxation Rules, 2011 11-3-2012 Refer Rule 4(a)(ii)[Service provided and invoice issued prior to change in rate of tax; only payment received after change.] 14-04-2012 Refer Rule 4(a)(i) [Only service provided before change; both issue of invoice and receipt of consideration after change of rate.] 31-03-2012 Refer Rule 4(a)(iii) [Service provided and payment received prior to change; only issue of invoice after change of rate of tax.] Total Service Tax Add : EC and SHEC @ 3% Total Service Tax Liability Rate of Service Service Tax Tax applicable (Rs.) 10% 50,000

B-2 18,00,000

12%

2,16,000

B-3 20,00,000

10%

2,00,000

4,66,000 13,980 4,79,980

Payment of Service Tax & Due Dates (P & S) 27

P.7 Solution :Bill No. B-001 Value of Taxable Value Reason for computing taxable value (Date of service service ( ` ) of service becoming taxable = 1-6-2012) 25,00,000 0 All payments received and invoice issued prior to 1-6-2012, hence, not taxable as per Rule 5(a) of the Point of Taxable Rules, 2011. Payments of Rs. 14,00,000 received and invoice issued prior to 1-6-2012, hence, not taxable as per Rule 5(a) of the Point of Taxation Rules, 2011. The balance consideration of Rs. 4,00,000 received on 10-06-2012 (i.e. on or after 1-6-2012 shall be liable to service tax). B-003 12,50,000 0 The payment has been received prior to 1-6-2012 and the invoice has been issued within the time-limit of 14 days. Hence, not liable to service tax as per Rule 5(b) of the Point of Taxation Rules, 2011. Liable to service tax, as the same is not covered by any of the clauses (a) or (b) of Rule 5 of the Point of Taxation Rules, 2011. Liable to Service tax.

B-002

18,00,000

4,00,000

B-004

22,00,000

22,00,000

B-005 Total

15,00,000

15,00,000 41,00,000 5,06,760

Tax @ 12.36%

P.8 All payments received and invoice issued prior to 1-9-2012, hence, not taxable as per Rule 5(a) of the Point of Taxation Rules, 2011. P.9 Rule 6(1) of Service Tax Rules, 1994. P.10 Who has paid Total Service Tax of Rs 10 lacs or above (including the amount of Service Tax paid by utilisation of CENVAT credit) in the preceding FY, shall deposit the Service Tax liable to be paid by him electronically, through internet banking. [Rule 6(2) of Service Tax Rules, 1994]. P.11 Rule 6(1) of the Service Tax Rules, 1994, inter alia, provides that service tax on the value of taxable services received by an individual during any quarter is payable by the 5th day of the month immediately following the said quarter. Therefore, in the given case, the due date for payment of service tax is 05/10/2012. Further, in case, if the assessee deposit the service tax by cheque, the date of presentation of cheque to the bank designated shall be deemed to be the date on which service tax has been paid subject to realization of that cheque. Thus, in this case, the date of payment will be 5th October 2012 as the cheque has been realized on 7th October, 2012. Since, the service tax has been paid on the due date, no interest and penalty is chargeable as there is no delay in payment of service tax.

Payment of Service Tax & Due Dates (P & S) 28

P.12 Solution :Advance Portion Adv. received towards all services in Sep. 2012 Amount billed for taxable services [2,10,000 - 70,000] Proportionate adv. received towards taxable services (60,000 x 1,40,000 / 2,10,000) Service tax @ 12% on ` 40,000 Add : Education Cess @ 2% Add : SHEC @ 1% Total Service Tax liability (Due Date 5.10.2012) Total Amount billed (February, 2013) Less : Amount for non-taxable Less : Amount already taxable Service Tax 1,00,000 12.36 100 (Due Date 5-3-2013) ` 60,000 1,40,000 40,000 4,800 96 48 4,944 2,10,000 70,000 1,40,000 40,000 1,00,000 12,360

Note : The Option provided in Rule 6(1) that the assessee may opt to pay service tax on receipt basis if the value of service provided in the preceding year is upto Rs. 50 lakhs is not available to a company. P.13 Solution :Value of services rendered (excluding service tax) ` 2,24,720 100 / 112.36 = ` 2,00,000 Amount received (excluding service tax) ` 1,68,540 100 112.36 = ` 1,50,000 Service tax payable - ` 1,50,000 12.36 100 = ` 18,540. Due date of deposit - 31.3.2013. Note : As per the point of Taxation Rules, 2011, the point of taxation is the date of invoice or date of payment, whichever is earlier. However, as per Rule 6(1), in case of individuals and partnership firms (including LLP) whose aggregate value of taxable services provided from one or more premises is Rs. 50 lakh or less in the previous financial year, the service provider shall have the option to pay tax on taxable services provided or to be provided by him up to a total of Rs. 50 lakhs in the current financial year, by the dates specified above with respect to the quarter in which payment is received. Assuming that this option under Rule 6(1) is availed, the service tax shall be payable on receipt basis. P.14 Rule 6(3) of Service Tax Rules, 1994 P.15 Solution :As per Point of Taxation Rules, 2011 the point of taxation is date of issue of invoice or the date of receipt of payment of the service, whichever is earlier. Since, here, the invoice had been raised first in order, hence, the service tax was payable with reference to the date of invoice. Accordingly, the service provider shall be liable to pay a service tax of Rs. 2,47,200 as indicated in the bill.

Payment of Service Tax & Due Dates (P & S) 29

If, in this case, it is assumed that the settlement referred to in (i) to (iii) above relates to re-negotiation due to deficient provision of service, or any terms contained in a contract, then, the service provider shall be eligible for credit of the excess service tax paid by him, which has been computed below : (i) The excess service tax, of which credit can be availed by the service provider, relatable to the amount adjusted against the bill = Rs. 22,47,200 22,00,000 = Rs. 47,200 (inclusive of service tax). Accordingly, the service tax credit under Rule 6(3) of Service Tax Rules = 47,200 12.36 112.36 = Rs. 5192 (ii) Service tax credit under Rule 6(3) of Service Tax Rules = 2,47,200 12.36 112.36 = Rs. 27,193 (iii) Service tax credit under Rule 6(3) of Service Tax Rules = 6,47,200 12.36 112.36 = Rs. 71,194. However, if the settlement of bill is due to any other reasons e.g. bad debts, etc., then, the benefit of Rule 6(3) shall not be available and the service tax shall continue to be ` 2,47,200. P.16 Rule 6(4) of Service Tax Rules, 1994.

Chapter - 4

Registration and Return


F Registration Section 69
v

Every person liable to pay the service tax under this chapter or the rules made there under shall, with in such time in such manner and in such form as may be prescribed make an application for registration to the Superintendent of Central Excise.

Rule 4 of Service Tax Rules, 1994


(1) Every person liable for paying the service tax shall v make an application to the Superintendent of Central Excise in v Form ST - 1 for registration v within thirty days from the date on which the service tax is levied. v provided that where a person commences the business of providing a taxable v service after such servie has been levied, v he shall make application for registration within thirty days from the date of such comencement. Procedure for Centralised Registration in case of Centralised Billing / Accounting System :- Where a person, liable for paying service tax on a taxable service, (i) Provides such service from more than one premises or offices; or (ii) receives such service in more than one premises or offices; or (iii) is having more than one premises or offices, which are engaged in relation to such service in any other manner, making such person liable for paying service tax, and has centralised billing system or centralised accounting system in respect of such service, and such centralised billing or centralised accountion systems are located in one or more premises, he may, at his option, register such premises offices from where centralised billing or centralised accounting systems are located. Where an assessee is providing more than one taxable service, he may make a single application, mentioning therein all the taxable services provided by him, to the concerned Superintendent of Central Excise. The Superintendent of Central Excise shall after due verification of the application form, grant a certificate of registration in Form ST-2 within seven days from the date of receipt of the application. If the registration certificates is not granted within the said period, the registration applied for shall be deemed to have been granted.

(2)

(3)

(4)

CA. Krishna Shankar Prasad, (9868308387)

Registration and Return [31]

(5)

where a registered assessee transfers his business to another person, the transferee shall obtain a fresh certifecate of registration. Every registered assessee, who ceases to provide the taxable service for which he is registered, shall surrender his registration certificate immediately.

(6)

F Furnishing of returns Section 70


Every person liable to pay the service tax shall himself assess the tax due on the services provided by him and shall furnish to the superintendent of Central Excise, a return in such Form and in such manner and at such frequency as may be prescribed.

Rule 7 of Service Tax Rules, 1994


(1) Half-yearly Return - Every assessee shall submit a half-yearly return in Form ST-3 (3 copies) alongwith a copy of the Form GAR-7. (2) Due Date for Return For the half year Ist April to 30th September Ist October to 31 March Due Date 25th October 25th April

Provided that the Form ST-3 required to be submitted by the 25th day of October, 2012 shall cover the period between 1st April to 30th June 2012 only. [w.e.f. 28-9-2012] CBEC has extended the date of submission of the return for the period 1-4-2012 to 30-6-2012 from 25-10-2012 to 25-11-2012 vide order no. 3/2012-S.T., dated 15-10-2012. Key Note :- If last day of payment and filing return is a public holiday, tax can be paid and return can be submitted on next working day.

Rule 7B of STR, 1994


An assessee may submit a revised return in Form ST - 3, in triplicate, to correct a mistake or omission, within a period of 90 days from the date of submission of the return under Rule 7.

CA. Krishna Shankar Prasad, (9868308387)

Registration and Return [32]

Rule 7C - Penalty for delay in furnishing return


Where the return prescribed under Rule 7 is not furnished within due date, the person liable to furnish the return shall pay to the Cenral Government, for the period of delay. Period of Delay, from the date of submission upto 15 days More than 15 days but upto 30 days More than 30 days ` 500 ` 1,000 ` 1,000 + ` 100 for each day in excess of 30 days. (Maximum ` 20,000) Provided also that where the gross amount of service tax payable is nil, the Central Excise officer may, on being satisfied that there is sufficient reason for not filing the return, reduce or waive the penalty. Amount of be paid

F Contents of the Return :The contents of the return inter alia, also include month-wise details of :
u u u u u u u u u u

amount received towards taxable service amount received in advance towards taxable service to be provided amount billed for exempted services and services exported without payment of tax amount billed for services on which tax is to be paid abatement claimed - value notification number of abatement and exemption service tax payable education cess & SHEC payable GAR-7 challan date and number Details of CENVAT Credit

Return in case of multiple service providers :- For an assessee who provides more than one taxable service, filing of a single return is sufficient. However, the details in each of the columns of the Form ST-3 have to be furnished separately for each of the taxable service rendered by him. Thus, instead of showing a lump sum figure for all the services together, service-wise details should be provided in the return.

CA. Krishna Shankar Prasad, (9868308387) F

Registration and Return [33]

E-Filing of Returns :- E-filing is a facility for the electronic filing of service tax returns by the assessee from his office, residence or any other place of choice, through the internet, by using a computer. E-Filing of return is an assessee facilitation measure of the department in continuation of its modernization and simplication program. It is an alternative to the manual filing of returns. This facility is available to all service providers.

E-filing of returns made mandatory for assessees :W.e.f. 1-10-2011 every assessee shall submit the half yearly return electronically.

Nil Return :- Even if no Service has been provided during a half year and no service tax is payable, the assessee has to file a Nil Return within the prescribd time limit.

Delayed return may be filed with late fee Section 70(1) :Every person liable to pay service tax shall himself assess the tax due on the services provided by him and shall furnish to the Superinendent of Central Excise, a return in such form and in such manner and at such frequency as may be prescribed. Section 70(1) to provide filing of periodical return after the due date with the prescribed late fee of not more than ` 20,000.

Scheme for Submission of Returns through Service Tax Prepares Section 71


(1) Without prejudice to the provisions of section 70, the Board may, by notification in the Official Gazette, frame a Scheme for the purposes of enabling any person or class of persons to prepare and furnish a retrn under section 70, and authorise a Service Tax Return Preparer to act as such under the Scheme. (2) A Service Tax Return Preparer shall assist the person or class of persons to prepare and furnish the return in such manner as may be specified in the Scheme framed under this section. (3) For the purposes of this section (a) Service Tax Return Preparer means any individual, who has been authorised to act as a Service Tax Return Preparer under the Scheme framed under this section; (b) person or class of persons means such person, as may be specified in the Scheme, who is required to furnish a return required to be filed under section 70. (4) The Scheme framed by the Board under this section may provide for the following, namely (a) the manner in which and the period for which the Service Tax Return Preparer shall be authorised,

CA. Krishna Shankar Prasad, (9868308387)

Registration and Return [34]

(b) the educational and other qualifications to be possessed, and the training and other conditions required to be fulflled, by a person to act as a Service Tax Return Preparer; (c) the code of conduct for the Service Tax Return Preparer; (d) the duties and obligations of the Service Tax Return Preparer; (e) the circumstances under which the authorisation given to a Service Tax Return Preparer may be withdrawn; (f) any other matter which is required to be, or may be, specified by the Scheme for the purposes of this section.

F Best Judgment Assessment Section 72 :If any person, liable to pay service tax (a) fails to furnish the return under section 70; (b) having made a return, fails to assess the tax in accordance with the provisions of this Chapter or rules made thereunder. the Central Excise Officer, may require the person to produce such accounts, documents or other evidence as he may deem necessary and after taking into account all the relevant material which is available or which he has gathered, shall by an order in writing, after giving the person an opportunity of being heard, make the assessment of the value of taxable service to the best of his judgment and determine the sum payable by the assesse or refundable to the assessee on the basis of such assessment.

F Special Audit Section 72A :(1) If the Commissioner of Central Excise, has reasons to believe that any person liable to pay service tax. (i) has failed to declare or determine the value of a taxable service correctly; or (ii) has availed and utilised credit of duty of tax paid (a) which is not within the normal limits having regard to the nature of taxable service provided, the extent of capital goods used or the type of inputs or input services used, or any other relevant factors as he may deem appropriate; and (b) by means of fraud, collusion, or any wilful misstatement or suppression of facts; and (iii) has operations spread out in multiple locations and it is not possible or practicable to obtain a true and complete picture of his accounts from the registered premises falling under the jurisdiction of the said Commissioner He may direct such person to get his accounts audited by a chartered accountant or cost accountant nominated by him, to the extent and for the period as may be specified by the Commissioner.

CA. Krishna Shankar Prasad, (9868308387)

Registration and Return [35]

(2)

The Chartered Accountant or cost accountant referred to in sub-section (1) shall, within the period specified by the said Commissioner, submit a report duly signed and certified by him to the said Commissioner mentioning therein such other particulars as may be specified by him. The provisions of sub-section (1) shall have effect notwithstanding that the accounts such person have been audited under any other law for the time being in force. The Person liable to pay tax shall be given an opportunity of being heard in respect of any material gathered on the basis of the audit under sub-section (1) and proposed to be utilised in any proceeding under the provisions of this Chapter or rules made thereunder.

(3)

(4)

Service tax collected from the recipient of service must be paid to The Central Government Section 73A :Every person, who is liable to pay service tax and has collected any amount in excess of the service tax assessed and paid on any taxable service from the recipient of taxable service in any manner as representing service tax, must pay the amount so collected immediately to the credit of the Central Government. This provision ensures that the service provider does not collect excess amount from the recipient of the service in the name of service tax.

Interest on Amount Collected in excess Section 73B :Where an amount has been collected, in excess of the tax assessed and paid, for any taxable service from the recipient of such service, the person who is liable to pay such amount shall, in addition to the amount, be liable to pay interest. The interest shall also be payable by the person who has collected any amount, which is not required to be collected as service tax. The interest shall be payable at the rate of 18% per annum from the first day of the month succeeding the month in which the amount ought to have been paid till the date of payment of such amount.

CA. Krishna Shankar Prasad, (9868308387)

Registration and Return [36]

Interest on late payment of Service Tax Section 75 :If the service tax is paid after the due date, simple interest is paid @ 18% p.a. for late payment of tax u/s 75 for the period of delay. This payment is mandatory and there is no provision to waive interest on delayed payment. Interest is paid for the period of delay and not for the whole month. If delay is for 10 days, interest will be paid for 10 days only. Period of delay is counted from the first day after the due date till the actual payment of service tax is made. Note : Rate of Interest for Small Scale Sector Assessee : 15% p.a. The term Small Scale Sector means a service provider whose value of taxable services does not exceed Rs. 60 lakh either during any of the years covered by the notice or during the last financial year.

Penalty for failure to make payment of service tax by due date Section 76 :Quantum of Penalty :- (A) Rs, 100 per day for every day during which such failure continues (B) At the rate of 1% of such tax, per month, [i.e. 12%p.a.] whichever is higher. However, total amount of penalty payable under this section shall not exceed 50% of the service tax that the assessee has failed to pay. Period for which penalty is to be levied :- Beginning with the first day after the due date and ending with the date of actual payment.

Penalty for non-registration Section 77(1)(a) :A person who is liable to pay service tax or is required to take registration, fails to take registration within stipulated time, shall be liable to pay a penalty of higher of the following two amounts: (a) Any amount upto Rs. 10,000. (b) 200 for every day during which failure continues.

F Role of a Chartered Accountants in the field of Service Tax consultancy


(i) Advisory Services (iii) Personal Representation
F

(ii) Procedural Compliance (iv) Certification and audit

New PAN Based Service Tax Code


The department has decided to introduce Service Tax Code based on PAN. Accordingly, it is compulsory for all concerns registered under the Service Tax Rules to obtain PAN whether or not they pay income tax. The Service Tax Code is a 15 digit alphanumeric code. First 10 digits will be 10 character PAN issued by Income Tax authorities. Next two characters will be ST. Last three will be numeric code 001, 002, 003, 004 etc. The concerned person should apply in prescribed form.

Registration and Return (P & S) 37

Problem and Solutions


P.1 Can Ramesh & Co., providing services from different locations and billing the clients from each location, opt for centralized service tax registration P.2 Mr. Govinda is a registered service provider. He transfers his business to Mr. Rohan on 31/8/2012. Explain the requirement to be complied with by Mr. Govinda and Mr. Rohan on such transfer under the provisions of Service Tax. P.3 What are the documents to be attached by a service provider alongwith an application for registration under service tax? P.4 What are the documents to be submitted along with the service tax return? P.5 What are the due dates for filiing service tax returns? P.6 Whether service tax return can be furnished after the due date? P.7 Who are the persons liable to file service tax return? P.8 Can service tax return be revised by a person? P.9 Is e-filling of service tax return permitted? P.10 Mr. Sohan is a multiple service provider and files only a single return. State with reasons whether he can do so? P.11 M/s Akash, a registered service provider did not render any services during the financial year 2012-2013. Whether he is required to file service tax return? P.12 Mr. Prakash has collected sum of ` 40,000 as a service tax from a client mistakenly, even though no service tax is chargeable for such service. Should the amount so collected be remitted to the credit of the central government? P.13 Mr. Mohit, an assessee, fails to pay service tax of Rs. 12 lakhs payable by 6-5-2012. Mohit pays the amount on 16-5-2012. Compute the amount of penalty payable by Mohit u/s 76. P.14 What is the EASIET scheme and state the benefits in the context of service tax?

Registration & Return (P & S) 38

[Solutions]
P.1 Rule 4(2) of Service Tax Rules, 1994 P.2 Requirement to be complied with by Mr. Govinda (The Transferor) : Every registered assessee, who ceases to provide the taxable service for which he is registered, shall surrender his registration certificate immediately. Requirement to be complied with by Mr. Rohan (The Transferee) :- He shall obtain a fresh certificate of registration. P.3 Documents to be submitted [Rule 4(1A)] : The CBEC may, by an order specify the documents which are to be submitted by the assessee alongwith the application within such period, as may be specified in the said order. (a) Copy of Permanent Account Number (PAN) (b) Proof of Residence (c) Constitution of the Applicant. (d) Power of Attorney in respect of authorised person(s). Application to the rejected if documents not submitted within 15 days : The above documents must be submitted to the concerned authority within a period of 15 days from the date of filing of the application for registration. Failure to do so would lead to rejection of the registration application. P.4 Along with ST-3 return following documents should be attached: (i) copies of GAR-7 challans (ii) a memorandum in form ST-3A giving full details of the difference between the amount of provisional amount of tax deposited and the actual amount payable for each month. FormST-3A is to be attached only when the assessee opts for provisional payment of service tax. P.5 Section 70 of Finance Act, 1994 & Rule 7 of Service Tax Rules, 1994. P.6 Delayed return may be filed with late fee. [Section 70(1)] [Rule 7C of Service Tax Rules, 1994] P.7 Person liable to file service tax return : Service provider who is liable to pay service tax. Service receiver who has been made liable to pay service tax under Service Tax Rules, 1994.

Registration & Return (P & S) 39

Service provider who is providing exempted services but who has provided services of value in excess of ` 9 lakhs [Service Tax (Registration of Special Categoery of Persons) Rules, 2005]. Input Service Distributor. P.8 Yes. [Rule 7(B) of Service Tax Rules, 1994]. P.9 Yes. P.10 Mr. Sohan who provides more than one taxable service, filing of a single return is sufficient. However, the details in each of the columns of the Form ST-3 have to be furnished separately for each of the taxable service rendered by him. Thus, instead of showing a lump sum figure for all the services together, service-wise details should be provided in the return. P.11 Yes. P.12 Mr. Prakash shall deposit the amount of ` 40,000 to the credit of Central Government. [Section 73A of Finance Act, 1994] P.13 In this case the default of failure to pay service tax has continued for 10 days. Hence, the penalty payable by Mohit shall be the higher of (a) 1% of the amount of default for 10 days = 1% 12,00,000 10 31 = ` 3,871 (b) Penalty calculated @ ` 100 per day for 10 days = ` 1,000. whicever is higher = 3871 but maximum limit of Penalty [50% of 12 L = 6 L] Hence, the penalty liable to be paid by Mohit is ` 3,871. P.14 EASIEST is an acronym for "Electronic Accounting System In central Excise & Service Tax (EASIEST)". EASIEST is one of the Computerization projects of CBEC. The concept of EASIEST is for on line transmission of Tax payment data from bank to Pay & Accounts office, Central Excise & Customs, Pune. It is aimed for obtaining a complete, correct and useful Tax Information Database at Tax Information Network (TIN) that can be used for accounting, reconciliation and for other MIS purpose. Advantages : Ease of operation and convenience Availability of the facility is on 24 7 basis On-line filling of challans and payment of taxes (No more queues and waiting) Instant Cyber Receipts for payment made Challans delivered at door step step and Service is normally free of charge.

Chapter - 5

Special Rate for Payment of Service Tax


F Life Insurance Service [Rule 6(7A) of the Service Tax Rule, 1994]
Means : An insurer or Re-insurer carrying on life insurance business. Options to Pay Service Tax : 3% of the gross premium charged in the first year and 1.5% of the gross premium charged in the subsequent years if the break-up of the gross premium into Risk Portion & Investment Portion is not given to the policy holder at the time of providing of service. Key Note :- The option shall be payable only in relation to composite policy (which consist of risk as well as saving components in insurance premium).

F Air Travel Agent (Relation to the Services of Booking of Tickets for Travel by Air) [Rule 6(7) of the Service Tax Rule, 1994]
Option given to the air travel agent in payment of Service Tax :u

Option to pay tax @ 0.60% of basic fare in case of domestic booking and @ 1.20% of basefare in case of international bookings. If the option is once exercised it cannot be changed during the financial year.

F Works contract Service :Composition Amount @ 4.8% of gross amount charged [works contract (Composition Scheme for payment of Service Tax) Rules, 2007]:

F Games of Chance Service [Rule 6(7C) of the Service Tax Rule, 1994]
Rate Rs 7000/- on every Rs 10 lakh (or part of Rs 10 lakh) of aggregate face value of lottery tickets printed by the organising State for a draw Rs 11000/- on every Rs 10 lakh (or part of Rs 10 lakh) of aggregate face value of lottery tickets printed by the organising State for a draw. Condition If the lottery scheme is one where the guaranteed prize payout is 80% or more. If the lottery or lottery scheme is one where the guaranteed prize payout is less than 80%.

Note : In case of Online Lottery, the aggregate face value of lottery tickets will be the aggregate value of tickets Sold. Note : : 2% 2% Education Education Cess Cess and and 1% 1% SHEC SHEC is is also also applicable applicable in in all all above above cases. cases. Note

CA. Krishna Shankar Prasad, (9868308387)

Spl. Rate for Payment of Service Tax [41]

F Money Changer
Determination of value of service in relation to money changing [Rule 2B of the Service Tax (Determination of Value) Rules, 2006; W.E.F. 1-4-2011] : Subject to the provisions of section 67, the value of taxable service provided, so far as it pertains to purchase or sale of foreign currency, including money changing, shall be determined by the service provider in the following manner :(i) for a currency, when exchanged from, or to, Indian Rupees (INR) : Value = Difference in the buying rate or selling rate and the RBI reference rate for that currency at that time Total units of currency. Example 1 Purchase of foreign currency by money-changer : US$1000 are sold by a customer at the rate of ` 45 per US$ RBI reference rate for US$ ` 45.50 at that time. The taxable value = (RBI Rate Buying Rate) No. of units of foreign currency bought = (` 45.5 ` 45) per US$ 1000 US$ = ` 500. Example 2 Sale of foreign currency by money-changer : INR 70000 is changed into Great Britain Pound (GBP) The exchange rate offered is ` 70, thereby giving GBP 1000. RBI reference rate at that time for GBP is ` 69. The taxable value = (Selling Rate RBI Rate) No. of units of foreign currency sold = (` 70 ` 69) per GBP 1000 GBP = ` 1,000. If no RBI rate available, Value = 1% of INR : In case where the RBI reference rate for a currency is not available, the value shall be 1% of the gross amount of Indian Rupees provided or received, by the person changing the money.

CA. Krishna Shankar Prasad, (9868308387)

Spl. Rate for Payment of Service Tax [42]

(b)

In case where neither of the currencies exchanged is Indian Rupee (i.e. One foreign currency is exchanged for another foreign currency) : Value shall be computed as follows Amount 1 = Foreign Currency sold RBI Reference Rate of that currency to Indian rupees at that time Amount 2 = Foreign Currency Bought RBI Reference Rate of that currency to Indian rupees at that time Gross Indian Rupees - Amount 1, or Amount 2, whichever is less Value = 1% of the Gross Indian Rupees xxx xxx xxx xxx

Money-changing services provided by a foreign exchange broker - Option to discharge service tax at specified percentage of gross amount of currency exchanged [Rule 6(7B) of Service Tax Rules, 1994, AMENDED W.E.F. 1-4-2012] : The person liable to pay service tax in relation to purchase or sale of foreign currency, including money changing, provided by a foreign exchange broker, including an authorised dealer in foreign exchange or an authorized moeny changer shall have the option to pay an amount calculated at the following rate towards discharge of his service tax liability instead of paying service tax at the rate specified in section 66 (i) 0.12% of the gross amount of currency exchanged for an amount upto ` 1,00,000, subject to the minimum amount of ` 30; and (ii) ` 120 + 0.06% of the gross amount of currency exchanged for an amount of rupees exceeding ` 1,00,000 and upto ` 10,00,000; and (iii) ` 660 + 0.012% of the gross amount of currency exchanged for an amount of rupees exceeding ` 10,00,000, subject to maximum amount of ` 6,000. Option once exercised to apply for that financial year and not to be withdrawn : The person providing the service shall exercise such option for a financial year and such option shall not be withdrawn during the remaining part of that financial year.

Special Rate for Payment of Service Tax (P & S) 43

Problem and Solutions


P.1 Whether life insurer carrying on the life insurance business has option to calculate service tax at some different rate? P.2 LIC of India provides you the following information for the month of December 2012. You are required to compute service tax payable by the company if the company has opted for composition option under Rule 6(7A) of the Service Tax Rules, 1994: (a) General policies : Total premiums collected ` 800 lakhs (Out of which 1st year premium is ` 200 lakhs) (b) Only Risk Cover Policies : Premiums collected ` 450 lakhs. (c) Variable Insurance Policies : Premiums collected ` 2,000 lakhs. (80% of the amount is allocated for investments on behalf of policy holder for which policy holder is given separate break up in premium receipts) P.3 M/s. Sky Airlines has sold tickets for transport of passengers to Japan during April, 2012. The total amount charged is Rs. 30 lakhs on the flight (200 tickets) of which Rs. 6 lakhs is towards passenger taxes. The airline paid a commission @ 16% of the basic fare, which is equal to the total amount charged by the airlines (net of government taxes), to an agent M/s. XYZ Associates who arranged for the booking of whole flight. M/s. XYZ Associates has passed over4% commission received by it to the customers i.e. only net 12% commission of the basic fare is retained by it as consideration towards services provided by it. Determine the service tax liability of M/s. XYZ Associates. Amounts are exclusive of service tax.

Special Rate for Payment of Service Tax (P & S) 44

P.4 M/s. Gulbahar Lotteries, established on 10-10-2012, is a distributor of lotteries organized by the State of Rajasthan. On account of festivals, two schemes of lotteries were announced as follows Particulars Total No. of tickets proposed under the scheme Face Value per ticket Value of guaranteed prize payouts Actual No. of tickets payouts Mode of conducting the scheme Super Dhamaka Bumper Dhamaka 10,60,000 100 70% 9,80,000 Printed (10,60,000 tickets) 80,000 500 90% 72,500 Online

Though M/s. Gulbahar Lotteries receives commission, however, it has learnt that there is some composition scheme under service tax law to discharge its service tax liability. If wishes to opt therefore tand asks you to compute its service tax liability thereunder. P.5 M/s. Haa-Haa Ho Ho Money Ltd., Delhi is an authorised money changer registered under FEMA, 1999. It has opted to discharge its service tax liability under Rule 6(7B) of the Service Tax Rules, 1994. It furnishes the following data of transactions of purchase and sale of foreign currency and requests you to compute its service tax liability therefrom : [April 2012] (i) 10 transactions involving currency exchange of ` 22,000 per transaction; (ii) 400 transactions involving currency exchange of ` 85,000 per transaction; (iii) 200 transactions involving currency exchange of ` 8,00,000 per transaction; (iv) One Transactions involving currency exchange of ` 25 lakh; and (v) 20 transactions involving currency exchange of ` 10 crore per transaction.

Special Rate for Payment of Service Tax (P & S) 45

[Solutions]
P.1 An insurer carrying on life insurance business shall have the option to pay tax : (i) on the gross premium charged from a policy holder reduced by the amount allocated for investment, or savings on behalf of policy holder, if such amount is intimated to the policy holder at the time of providing of services; (ii) in all other cases, 3% of the premium charged from policy holder in the 1st year and 1.5% of the premium charged from policy holder in the subsequent years; [w.e.f. 1-4-2012] towards the discharge of his service tax liability instead of paying service tax at the rate specified in section 66B of the Finance Act, 1994. However, such option shall not be available in cases where the entire premium paid by the policy holder is only towards risk cover in life insurance. P.2 Sol.: Computation of service tax payable by the insurance company Taxable Tax Service Value Tax Rate (` in lakhs) (` in lakhs)

Particulars General policies : (i) First year premium (ii) Subsequent years i.e. policies issued in earlier years Only Risk Cover Policies : Since the entire premium is for risk cover, hence, the option under Rule 6(7A) is not available and normal tax rate of 12% applies. Variable Insurance Policies : (80% of the amounts allocated for investments on behalf of policy holder for which policy holder is given separate break up in premium receipts) Premium charged Amount allocated towards investment i.e. 80%. Thus, the taxable value shall be 20% and tax rate shall be normal rate of 12%. Total Add : EC and SHEC @ 3% Total Service Tax

200

3%

6 9

600 1.5%

450

12%

54

400

12%

48 117 3.51 120.51

Special Rate for Payment of Service Tax (P & S) 46

P.3 Sol.: The relevant computations are shows hereinbelow Service Tax payable by M/s. XYZ Associates Basic Fare (Total 30 lakhs Government Taxes 6 lakhs] Commission received from the airlines (it forms part of the value) [Basic Fare 16%] [The commission received by M/s. XYZ from airlines is wholly liable to service tax. No deduction can be allowed in respect of commission passed on to customers.] Service Tax on Taxable value @ 12% [A] 24,00,000

3,84,000 46,080 28,800 28,800 864 29,664

Option to pay service tax @ 1.2% of the basic fare on international bookings under Rule 6(7) of the Service Tax Rules, 1994 [B] Service Tax liability [Lower of [A] or [B]] Add.: EC and SHEC @ 3% [EC and SHEC are imposable even on optional amount payable under Rule 6(7) of Service Tax Rules, 1994] Total service tax payable

P.4 Solution :- M/s. Gulbahar Lotteries can very well opt for the composition scheme for payment of service tax. The service tax liability shall be as under Particulars Total No. of tickets (In case of Super Dhamaka scheme, the total no. of tickets printed shall be taken; while in case of Bumper Dhamaka, since the lottery is conducted online, hence, the actual no. of tickets sold shall be taken) Face Value per ticket Aggregate face value of lottery tickets Value of guaranteed prize payouts Service Tax payable (for every ` 10 lakhs or part thereof) No. of Units of ` 10 lakhs or part thereof [Any part of ` 10 lakhs shall be fully covered] Service Tax payable Total Service Tax Add : EC and SHEC @ 3% Total Service Tax Liability Super Dhamaka Bumper Dhamaka 10,60,000 72,500

100 10,60,00,000 70% 11,000 106 11,66,000

500 3,62,50,000 90% 7,000 37 2,59,000 14,25,000 42,750 14,67,750

Special Rate for Payment of Service Tax (P & S) 47

P.5 The service tax liability under Rule 6(7B) of the Service Tax Rules, 1994 is as follows: Currency Exchanged per transaction Service Tax per transaction Computation 0.12% of currency exchanged, subject to the minimum amount of ` 30. 0.12% of currency exchanged, subject to the minimum amount of ` 30. ` 120 + 0.06% of the gross amount of currency exchanged for amount of rupees exceeding ` 1,00,000. ` 660 + 0.012% of gross currency exchanged for sum exceeding ` 10,00,000, subject to maximum of ` 6,000. ` 660 + 0.012% of gross currency exchanged for sum exceeding ` 10,00,000, subject to maximum of ` 6,000. Total Add : EC and SHEC @ 3% Total Service Tax Liability Amount ` 30 102 540 No. of Total transService Tax actions

22,000 85,000 8,00,000

10 400 200

300 40,800 1,08,000

25,00,000

840

840

10,00,00,000

6,000

20

1,20,000

2,69,940 8098 2,78,038

Chapter - 6

Exemption from Service Tax


(I) Exemption to Small Scale Service Provider [N.No. 33/2012 S.T.]
Small Scale Service Provider :The aggregate value of taxable services rendered by whom, from one or more premises, does not exceed ` 10 lakh in the preceding financial year. Exemption available to small scale service provider :- Exemption shall be operative only for aggregate value not exceeding ` 10 lakh in any financial year. If the aggregate value in any financial year exceeds ` 10 lakhs, then such excess over ` 10 lakhs shall be chargeable to service tax. Key Note :Aggregate value : Aggregate value means (i) the sum total of value of taxable service charged in the first consecutive invoces issued or required to be issued, as the case may be, during a financial year. (ii) but does not include value charged in invoices issued towards such services which are exempt from whole of service tax leviable thereon under Section 66B of the Act.

Service Tax (Registration of Special Category of Persons) Rules, 2005


Following will require registration as per the Rules (i) Input Service Distributors - within thirty days of the commencement of business. (ii) Service provider who is exempt but whose aggregate value of taxable service exceeds ` 9 lakhs . They have to apply for registrationin form ST-1 - within thirty days of exceeding the aggregate value of taxable service of 9 lakh rupees. The provisions of rule 4 of Service Tax Rules, 1994 shall be applicable to such persons.

(II) Exemption to Services for Official or Personal use of Foreign Diplomatic Mission [Notification No. 27/2012-ST]
The taxable services provided by any person (a) for the official use of a foreign diplomatic mission or consular post in India, or (b) for personal use or for the use of the family members of diplomatic agents or career consular officers posted therein are exempt, subject to certain conditions.

CA. Krishna Shankar Prasad, (9868308387)

Exemption from Service Tax [49]

(III) Exemption to Services Provided by TBI/STEP (Incubator) [N.No. 32/2012-ST]


The taxable services, provided or to be provided, by (a) a Technology Business Incubator (TBI) or (b) a Science and Technology Entrepreneurship Park (STEP) recoginzed by National Science and Technology Entrepreneurship Development Board (NSTEDB) of Department of Science and Technology, Government of India, are exempt, subject to condition that TBI/STEP is to furnish prescribed information to the Assistant Commissioner or the Deputy Commissioner of Central Excise before availing the exemption by 30th June of each financial year.

(IV) Rebate of Service Tax paid in Taxable Services received by an Exporter of Goods and used for Export of Goods [Notification No. 41/2012-ST]
The Central Government has granted rebate of service tax paid on the taxable services which are received by an exporter of goods and used for export of goods, as follows (1) Services eligible Specified Services : Specified services means (a) in the case of excisable goods, taxable services that have been used beyond the place of removal, for the export of said goods; (input services availed upto the place of removal are eligible as input service for CENVAT Credit under CENVAT Credit Rules) (b) in the case of other goods, taxable services used for the export of said goods (2) Time-limit for claim : 1 year from the date of export of the said goods.

CA. Krishna Shankar Prasad, (9868308387)

Exemption from Service Tax [50]

(V) Exemption to developer of, or units located in, Special Economic Zone. Notification No. 40/2012-S.T.] The exemption available to developer of, or units located in, a Special Economic Zone (SEZ) is as follows (a) Exemption on services received by SEZ for use in SEZ : The taxable services (i) received by a unit located in a SEZ or Developer of SEZ and (ii) used for the authorised operations. are exempt from service tax. Only specified services are eligible : Only the list of services as are required for the authorised operations and approved by Approval Committee of concerned SEZ are eligible for exemption. (b) Exemption to be by way of refund, except in case of services wholly consumed within SEZ : The exemption shall be provided by way of refund of service tax paid on the specified services (i.e. services for which approval has been obtained) received for the authorised operations in a SEZ. However, where the specified services received and used for authorised operations are wholly consumed within the SEZ, the provider of such services or the receiver of such services on reverse charge basis, as the case may be, has the option not to pay the service tax. (c) Proportionate refund when services not wholly consumed within SEZ : Where the specified services received by Unit or Developer, are not wholly consumed within SEZ, i.e., shared between authorised operations in SEZ Unit and Domestic Tariff (DTA) Unit, refund shall be restricted as follows

Service Tax paid on specified services Maximum = used for SEZ Authorised Operations Refund Total Turnover for the period shared with DTA unit for the period
(A) Total turnover means the sum total of the value of all output services and exempted services provided, including, the value of services exported, bought out goods sold, during the period to which the invoices pertain and the exporter claims the facility of refund under this notification. (B) Turnover of SEZ Unit shall mean the sum total of the value of final products and output services exported during the period of which the invoices pertain and the exporter claims the facility of refund under this notification. (d) No CENVAT credit of specified services on which refund claimed : In order to avail this exemption, no CENVAT credit of service tax paid on the specified services used for the authorised operations in a SEZ must been taken under the CENVAT Credit Rules, 2004. SEZ units liable to pay service tax on non-exempt services (Circular No. 105/08/2008 dtd. 16-9-2008]: Section 66 does not exclude SEZs from scope of its levy. Therefore, service tax is applicable on taxable services provided by SEZ units, except such services which are exempt as aforesaid. Accordingly, SEZ units, providing taxable services to any person for consumption in Domestic Tariff Area (DTA) or providing any taxable service which is otherwise not exempt, are liable to pay service tax.

Export Turnover of SEZ Unit for the period

CA. Krishna Shankar Prasad, (9868308387)

Exemption from Service Tax [51]

(VI) Negative List of Services, Section 66D (w.e.f. 1-7-2012) These Services are not chargeable to Service Tax. The negative list is given below (1) services by Government or a local authority excluding the following services to the extent they are not covered elsewhere (i) services by the Department of Posts by way of speed post, express parcel post, life insurance and agency services provided to a person other than Government. (ii) services in relation to an aircraft or a vessel, inside or outside the precincts of a port or an airport. (iii) transport of goods or passengers or (iv) support services, other than services convered under clauses (i) to (iii) above provided to business entities (2) services by the Reserve Bank of India. (3) services by a foreign diplomatic mission located in India. (4) services relating to agriculture or agricultural produce by way of (i) agricultural operations directly related to production of any agricultural produce including cultivation, harvesting, threshing, plant protection or seed testing. (ii) supply of farm labour. (iii) processes carried out at an agricultural farm including tending, pruning, cutting, harvesting, drying, cleaning, trimming, sun drying, fumigating, curing, sorting, grading, cooling or bulk packaging and such like operations which do not alter the essential characteristics of agricultural produce but make it only marketable for the primary market. (iv) renting or leasing of agro machinery or vacant land with out without a structure incidental to its use. (v) loading, unloading, packing, storage or warehousing of agricultural produce. (vi) agricultural extension services. (vii) services by any Agricultural Produce Marketing Committee or Board or services provided by a commission agent for sale or purchase of agricultural produce. (5) trading of goods. (6) any process amounting to manufacture or producting of goods. (7) selling of space or time slots for advertisements other than advertisements broadcast by radio or television. (8) service by way of access to a road or a bridge on payment of toll charges. (9) betting, gambling or lottery. (10) admission to entertainment events or access to amusements facilities.

CA. Krishna Shankar Prasad, (9868308387)

Exemption from Service Tax [52]

(11) transmission or distribution of electricity by an electricity transmission or distribution utility. (12) services by way of (i) pre-school education and education up to higher secondary school or equivalent. (ii) education as a part of a curriculum for obtaining a qualification recognised by any law for the time beign in force. (iii) education as a part of an approved voacational education course. (13) services by way of renting of residential dwelling for use as residence. (14) services by way of (i) extending deposits, loans or advances in so far as the consideration is represented by way of interest or discount. (ii) inter se sale or purchase of foreign currency amongst banks or authorised dealers of foreign exchange or amongst banks and such dealers. (15) service of transportation of passengers, with or without accompanied belongings, by (i) a stage carriage (ii) railways in a class other than (A) first class or (B) an airconditioned coach (iii) metro, monorail or tramway (iv) inland waterways (v) public transport, other than predominantly for tourism purpose, in a vessel between places located in India and (vi) metered cabs, radio taxis or auto rickshaws (16) services by way of transportation of goods (i) by road except the services of (A) a goods transportation agency or (B) a courier agency (ii) by an aircraft or a vessel from a place outside India up to the customs station of clearance in India or (iii) by inland waterways (17) funeral, burial, crematorium or mortuary services including tranportation of the deceased.

Exemption from Service Tax (P & S) 53

Problem and Solutions


P.1 Who is Small Scale Service Provider? P.2 On April 1, 2012, CA. Anil Kumar Agrawal is unregistered service provider. He provides the following details in respect of taxable services provided during the financial year 2012-13: Date of transaction Particulars Amount of bill (excluding service tax) Payment received `

June 30, 2012 July 14, 2012 September 30, 2012 September 30, 2012 December 31, 2012 December 31, 2012 January 31, 2013 January 31, 2013 March 31, 2013 March 31, 2013

Advance received from a customer (on July 14, 2012 Bill No. 001 was issued) Bill No. 001 issued Bill No. 002 issued Payment received against Bill No. 002 Bill No. 003 issued (for taxable services rendered during December 2012) Money received against Bill No. 003 Bill No. 004 issued (for taxable services rendered during January 2013) Money received against Bill No. 004 Bill No. 005 issued (for taxable services rendered during March 2013) Money received against Bill No. 005 2,50,000 3,00,000 2,00,000 1,00,000 4,00,000

1,00,000 4,00,000 2,00,000 3,00,000 2,50,000

The service tax provider complies with the provisions of registration and collection of service tax as per servie tax laws. He gets registered during the financial year. Compute the service tax liability of service provider for the year 2012-13 considering the rate of service tax @ 12.36%. P.3 Discuss the accountability of an input service distributor who may not be liable to pay service tax. P.4 When does a small service provider require to register under the Service Tax Act, but not liable to collect and pay Service Tax?

Exemption from Service Tax (P & S) 54

P.5 Discuss whether the following person are liable to apply for registration under the service tax law and if yes, by which date -(a) A provider of taxable service, whose aggregate value of taxable services is ` 7,80,000 upto 31.3.2013. (b) A provider of taxable service, whose aggregate value of taxable services is ` 9,01,000 on 1.12.2012. (c) A provider of taxable service, who has provided services as follows : Aggregate value of services upto 31.5.2012 (i.e. before the service became taxable) ` 6,00,000. Aggregate value of taxable services from 1.6.2012 to 31.3.2013 ` 7,95,000. (d) A provider of taxable service who starts his business w.e.f. 11.8.2012 and whose aggregate value of taxable services as on 10.10.2012 becomes ` 9,02,000. P.6 Mr. Fardeen, the owner of a property had entered into an agreement with a bank. The agreement was entered into on 1.4.2012 to give ground floor of the property on rent on monthly rent of ` 75,000 p.m. The bank had taken the property for commercial purpose. Explain whether Mr. Fardeen is liable to pay service tax on the transaction with bank ? P.7 Whether export service provided by Service Provider is excluded for the purpose of Payment of Service Tax? P.8 A partnership firm, gives the following particulars relating to the services provided to various clients by them for the half-year ended on 30.9.2012 : (i) Total bills raised for ` 13,75,000 (excluding service tax) out of which bill for ` 75,000 was raised on an approved International Organisation and payments of bills for ` 1,00,000 were not, received till 30.9.2012. Amount of ` 50,000 was received as an advance from XYZ Ltd. on 25.9.2012 to whom the services were to be provided in October, 2012. are required to work out the : Taxable value of services Amount of Service Tax payable.

(ii) You (a) (b)

Note : The value of services provided by firm during the preceding year was ` 60 lakhs. P.9 XYZ, a unit in SEZ, received services from various service providers in relation to authorized operations in SEZ during the month December, 2012. At the time of making payment, service provider asks it to pay tax, however, it argues that service tax is not applicable on taxable services provided to it. The following details are furnished for the month December, 2012: (i) Value of Taxable services wholly consumed within SEZ : ` 8,00,000

Exemption from Service Tax (P & S) 55

(ii) Value of Taxable services not wholly consumed within SEZ but exclusively used for authorized operations in SEZ : ` 28,00,000 (iii) Value of Taxable Services Shared between SEZ units and DTA units : ` 12,00,000 (iv) Value of Taxable Services used wholly for DTA units : ` 9,00,000 (v) Export Turnover of SEZ unit : ` 3,00,00,000 (vi) Total Turnover : ` 5,00,00,000 (vii) Service Tax Rate : 12.36% Compute the amount payable as service tax by M/s. XYZ along with incentives available to it by way of exemption / refund / CENVAT Credit for the month assuming that all conditions are complied with. P.10 Define the following with reference to Finance Act, 1994 (a) Approved Vocational Educaiton Course. (b) Business Entity. (c) Works Contract.

Exemption from Service Tax (P & S) 56

[Solutions]
P.1 Small Scale Service Prodvider :The aggregate value of all taxable services rendered by a provider of taxable service from one or more premises, does not exceed Rs. 10 lakhs in preceding Financial Year. Exemption available to small scale service provider :In Financial Year, the First Receipts of Rs. 10 lakh shall be exempt from payment of whole of service tax. Aggregate Value means (i) the sum total of value of taxable services charged in the first consecutive invoices issued or required to be issued, as the case may be, during a financial year (ii) but does not include value charged in invoices issued towards such services which are exempt from whole of service tax leviable thereon under any other notification. P.2 Computation of service tax liabilityTotal amount receipt ` 12,50,000

Less exemption available to Small Service Provider 10,00,000 Taxable Value for Service Tax Service Tax @ 12.36% 2,50,000 30,900

Notes : CA. Anil Kumar Agrawal is small service provider during the financial year 2012-2013 and therefore he is entitled to claim exemption of ` 10 lakhs. P.3 Input Service Distributor :Registration :- Input Service Distributor will require registration as per Service Tax (Registration of Special category of persons) Rules, 2005 within thirty days of the commencement of business. [Registration Form ST-1] Return :- Input Service Distributor shall also file return showing the credit availed and credit distributed by it. P.4 Service Tax (Registration of Special Category of Person) Rules, 2005.

Exemption from Service Tax (P & S) 57

P.5 Solution :(a) Not liable for registration, as aggregate value of taxable services does not exced ` 9 lakhs. (b) Liable for registration, application to be made upto 31.12.2012. (c) Not liable for registration. (d) Liable for registration, application to be made upto 9.11.2012. P.6 Solution :The total value of service provided durin the financial year is ` 9,00,000. In case Mr. Fardeen is not providing any other taxable services, there shall be no liability to pay service tax as the value of services provided during the year is less than ` 10,00,000. Note : It is assumed that Mr. Fardeen is eligible for small service provider exemption during the year 2012-13, as the value of services provided by him during 2011-12 doesn't exceed ` 10 lakhs. P.7 Export of Services [Rules 6A of Service Tax Rules, 1994] P.8 Solution :` 13,75,000 75,000 13,00,000 50,000 13,50,000 1,66,860

Total bills raised Less : Bills raised for an approved international organisation (not liable to service tax) Add : Amount received in Advance Service tax payable 13,50,000 12.36 100 =

Note : As per the Point of Taxation Rules, 2011, the point of taxation is date of invoice or the date of receipt of payment, whichever is earlier. In this case, the bill has already been raised during 30-9-2012, while the payment is received after that date, accordingly, the point of taxation lies on 30-9-2012 itself.

Exemption from Service Tax (P & S) 58

P.9 Solution :S. No. (i) Particulars Taxable services wholly consumed within SEZ Value 8,00,000 Service Tax payable, @ 12.36% Fully Exempt; Option not to pay service tax 3,46,080 Refund, if any No need to claim refund, as tax not paid 3,46,080

(ii) Taxable services not wholly consumed but 28,00,000 exclusively used for SEZ Exemption not available as not wholly consumed, but, 100% refund available. (iii) Shared Services i.e. services shared 12,00,000 between SEZ and DTA i.e. Proportionate Refund = Service Tax Export Turnover ` 300 lakh Total Turnover ` 500 lakh (iv) Services used wholly for DTA No refund / exemption Total Amount available as CENVAT credit to the DTA units [Service Tax payable Refund] [Proportionate amount of service tax paid on shared services that have not been refunded after applying the formula shall be available to the DTA Units of the entity as CENVAT Crtedit.] 9,00,000

1,48,320

88,992

1,11,240 6,05,640

4,35,072 1,70,568

P.10 Solution : (a) Approved Vocational Educaiton Course [Section 65B(11)] "Approved vocational eudcation course" means (i) a course run by an industrial training institute or an industrial center affiliated to the National Council for Vocational Training offering courses in designated trades notified under the Apprentices Act, 1961; or (ii) a Modular Employable Skill Course, approved by the National Council of Vocational Training, run by a person registered with the Directorate General of Employment and Training, Union Ministry of Labour and Employment; or (iii) a course run by an institute affiliated to the National Skill Development Corporation set up by the Government of India. (b) Business Entity [Section 65B(17)] "Business Entity" means any person ordinarily carrying out any activity relating to industry, commerce or any other business or profession.

Exemption from Service Tax (P & S) 59

(c) Works Contract [Section 65B(54)] "Works Contract" means a contract wherein transfer of property in goods involved in the execution of such contract is leviable to tax as sale of goods and such contract is for the purpose of carrying out construction, erection, commissioning, installation, completion, fitting out, repair, maintenance, renovation, alteration of any movable or immovable property or for carrying out any other similar activity or a part thereof in relation to such property.

You might also like